Sunteți pe pagina 1din 117

No -7 Set/2008 ISSN 1518-6075

revista

DA OLIMP IADA

OLIMP IADA DE MATEMATICA DO ESTADO DE GOIAS

UNIVERSIDADE FEDERAL DE GOIAS INSTITUTO DE MATEMATICA E ESTATISTICA

Dados Internacionais de Cataloga c ao da Publica c ao(CIP) (GPT/BC/UFG) Revista da Ol mpiada/Universidade Federal de Goi as/ Instituto de Matem atica e Estat stica. No - 7 (jan.2007/dez. 2008). Goi ania: Editora da UFG, 2008-v. Anual. Matem atica - Peri odicos - ISSN 1518-6075 - CDU: 51(05) Comit e Editorial. Jos e Hil ario da Cruz, Rog erio Queir oz Chaves, Ronaldo Alves Garcia. Editora convidada para este n umero: Edm eia Fernandes da Silva. Editora c ao Jos e H. da Cruz Tiragem 2.500 exemplares Arte da Capa Leonardo M. Pel a Postagem 2o - semestre de 2008

Revista da Olimp ada, no - 7, 2008 Universidade Federal de Goi as Instituto de Matem atica e Estat stica Campus Samambaia Caixa Postal 131 74.001-970 - Goi ania - Goi as Tel.: (62) 3521 1208, Fax: (62) 3521 1180 Vers ao eletr onica dispon vel em: www.ime.ufg.br Os artigos assinados s ao da responsabilidade dos autores. E permitida a reprodu ca o, desde que seja citada a fonte.

Revista da Olimp ada - IME - UFG, no - 7, 2008

Apresenta c ao
Caro Leitor, A Revista Olimp ada de Matem atica do Estado de Goi as e uma publica c ao anual do Instituto de Matem atica e Estat stica da UFG e tem como principal p ublico alvo, professores e estudantes do ensino fundamental e m edio. Tem como meta ser um ve culo de: difus ao cultural, integra ca o Universidade/Escola, espa co de cria ca o e reex ao cr tica sobre a ci encia Matem atica. Esperamos que, na leitura dos artigos e problemas propostos e resolvidos, o leitor fa ca anota co es complementares, amplie seus conhecimentos nas bibliograas citadas e principalmente, seja capaz de difundir oralmente e com naturalidade o conte udo assimilado transmitindo-o a seus colegas, amigos, pais, lhos, etc. Tamb em gostar amos de receber sugest oes e problemas que ser ao submetidos a an alise para poss vel publica c ao. Acreditamos que o dom nio da ci encia, em particular da matem atica, e o seu bom uso s ao fundamentais para o desenvolvimento da humanidade e nossa aten c ao para este fato e que todos possam apreciar, aqui, a riqueza da matem atica e sejam agentes transformadores para elevarmos a cultura matem atica no nosso Estado e no nosso Pa s.

Goi ania, 17 de setembro de 2008 Os Editores.

Universidade Federal de Goi as


Edward Madureira Brasil Reitor Benedito Ferreira Marques Vice-Reitor Sandramara Matias Chaves Pr o-Reitora de Gradua ca o Divina das Dores de Paula Cardoso Pr o-Reitora de Pesquisa e Gradua ca o Orlando Afonso Valle do Amaral Pr o-Reitor de Administra ca o e Finan cas Jeblin Ant onio Abra ao Pr o-Reitor de Desenvolvimento Institucional e Recursos Humanos Anselmo Pessoa Neto Pr o-Reitor de Extens ao e Cultura Ernando Melo Filizzola Pr o-Reitor de Assuntos da Comunidade Universit aria Gisele de Ara ujo Prateado Gusm ao Diretora do Instituto de Matem atica e Estat stica

Olimp ada de Matem atica do Estado de Goi as Comiss ao Organizadora (ano 2006) Rog erio Queir oz Chaves (coordenador), Luciana Maria Dias de Avila Rodrigues e F abio Vitoriano e Silva. Comiss ao Organizadora (ano 2007) Rog erio Queir oz Chaves (coordenador), Edm eia Fernandes da Silva.
Universidade Federal de Goi as - Instituto de Matem atica e Estat stica Campus Samambaia - Caixa Postal 131 - CEP 74.001-970 - Goi ania-GO Correio eletr onico: omeg@mat.ufg.br Tel:(62)3521-1208 Fax:(62)3521-1180 Site: www.ime.ufg.br/extensao/olimpiada

Olimp ada de Matem atica do Estado de Goi as

Indice
Colet anea de Problemas Solu co es da Colet anea de Problemas Classicados na XIV OMEG N vel 1 . . . . . . . . . . . . . N vel 2 . . . . . . . . . . . . . N vel 3 . . . . . . . . . . . . . Classicados na XV N vel 1 . . . . . . . N vel 2 . . . . . . . N vel 3 . . . . . . . . . . 1 8

2005 20 . . . . . . . . . . . . . . . . . . 20 . . . . . . . . . . . . . . . . . . 21 . . . . . . . . . . . . . . . . . . 22 24 24 25 27

OMEG - 2006 . . . . . . . . . . . . . . . . . . . . . . . . . . . . . . . . . . . . . . . . . . . . . . . . . . . . . . . . . . . . . . . . . . . . . . . . . . . . . .

Classicados na XVI OMEG N vel 1 . . . . . . . . . . . . . N vel 2 . . . . . . . . . . . . . N vel 3 . . . . . . . . . . . . . Not cias Solu co es Comentadas N vel 1 . . . . . . . . N vel 2 . . . . . . . . N vel 3 . . . . . . . . Solu co es Comentadas N vel 1 . . . . . . . . N vel 2 . . . . . . . . N vel 3 . . . . . . . .

2007 28 . . . . . . . . . . . . . . . . . . 28 . . . . . . . . . . . . . . . . . . 30 . . . . . . . . . . . . . . . . . . 31 34

das Provas XIV . . . . . . . . . . . . . . . . . . . . . . . . . . . . . . . . .

OMEG . . . . . . . . . . . . . . .

. . .

2005 . . . . . . . . . . . . . . . . . . . . .

36 36 40 43

das Provas XV OMEG - 2006 49 . . . . . . . . . . . . . . . . . . . . . . . . 49 . . . . . . . . . . . . . . . . . . . . . . . . 53 . . . . . . . . . . . . . . . . . . . . . . . . 57

Olimp ada de Matem atica do Estado de Goi as

ii OMEG . . . . . . . . . . . . . . . . . . 2007 . . . . . . . . . . . . . . . . . . . . . 62 62 66 70 77 84 90

Solu co es Comentadas N vel 1 . . . . . . . . N vel 2 . . . . . . . . N vel 3 . . . . . . . .

das Provas XVI . . . . . . . . . . . . . . . . . . . . . . . . . . . . . . . . .

O Teorema das Quatro Cores Inteiros Gaussianos e C alculo do Valor de Calculando Logaritmos de uma Forma Eciente

Progress oes - Uma Atividade de Introdu c ao ao Conceito de Limite 95 N umeros Perfeitos e Primos de Mersenne 99

Revista da Olimp ada - IME - UFG, no - 7, Setembro 2008,

1-19

Colet anea de Problemas

W erica Pricylla de Oliveira Valeriano Edm eia Fernandes da Silva

Resumo. Nesta se c ao apresentamos uma colet anea de problemas destinados a alunos e professores interessados n ao apenas em se prepararem e prepararem seus alunos para participar das Olimp adas de Matem atica que ocorrem no Brasil e no mundo, mas sim para aqueles que gostam da matem atica e que gostam de um desao. 1. Maria escolhe 8 n umeros inteiros e os escreve nos v ertices de um cubo. Em cada aresta ela escreve a soma dos n umeros que colocou nos seus extremos. Em cada face ela escreve a soma dos n umeros que colocou nos v ertices mais os n umeros que colocou nas arestas que formam esta face. e poss vel escolher oito n umeros de forma que: a) todas as faces tenham valor igual a 100? b) a soma de todos os 26 n umeros seja 1995? 2. Dois homens caminhavam no deserto. Um deles possuia 5 litros de agua e 5 p aes e o outro trazia 3 litros de agua e 3 p aes. No momento em que se preparavam para descansar, avistaram um homem que estava bastante exausto e com sede. Resolveram repartir a agua e os p aes igualmente entre os tr es. Dois dias depois chegaram a um o asis. Ao se despedir, em sinal de agradecimento, o homem deu 8 moedas de ouro para os dois que tinham salvo a sua vida. Se a divis ao foi feita de forma justa, com quantas moedas cada um deles cou? 3. Na sua festa de anivers ario, o jovem Guilherme resolveu fazer a seguinte brincadeira com os seus 5 amigos mais chegados: ele daria sua cole c ao de bolinhas de gude a quem adivinhasse de que cores eram duas
1

Graduanda em Matem atica/Bolsista/PROEC/UFG, 2006.

Olimp ada de Matem atica do Estado de Goi as

bolas que ele havia escondido numa caixa. Ele explicou que as duas bolas tinham cores diferentes, dentre as 6 cores a seguir: verde, amarela, azul, branca, vermelha e preta. Seus amigos deram os seguintes palpites, respectivamente: verde e branca; amarela e branca; amarela e vermelha; azul e vermelha; branca e preta. Guilherme, chateado, comunicou a seus amigos que um deles havia errado as duas cores e os outros tinham acertado uma mas errado a outra. De repente, teve uma id eia luminosa: daria sua cole c ao a quem, a partir das suas informa c oes, descobrisse as cores das bolas. Carlos Gustavo, que n ao era bom de adivinha c ao, mas gostava de quebra-cabe cas, deu logo a resposta certa. Mostre como ele descobriu as cores das bolas na caixa. 4. Na gura o n umero 8 foi obtido somando-se os dois n umeros diretamente abaixo de sua casinha. Fazendo-se o mesmo para preencher as casas em branco, obt em-se o 42 na casa indicada. Qual o valor de x?

42

8 3 5 x 6

5. Em um quadrado m agico, a soma dos 3 n umeros de cada linha, coluna ou diagonal e sempre a mesma. A seguir temos um quadrado m agico, parcialmente preenchido. Qual o valor de x?

6. Em lia inventou uma brincadeira. Escreveu alguns algarismos na primeira linha de uma folha. Depois, na segunda linha, fez a descri c ao dos algarismos digitados da seguinte maneira: ela apresentou as quantidades de cada um dos que aparecem, em ordem crescente de algarismos. Por exemplo, ap os digitar 21035662112, ela digitou 103132131526, pois em 21035662112 existe um algarismo 0, tr es algarismos 1, tr es algarismos 2,

Olimp ada de Matem atica do Estado de Goi as

um algarismo 3, um algarismo 5 e dois algarismos 6. Em lia gostou tanto de fazer isso que decidiu preencher v arias folhas com essa brincadeira, come cando com 01 na primeira linha da primeira folha. Quais s ao os dois primeiros algarismos da esquerda do que ela digitou na 2006a - linha? 7. A minha calculadora est a avariada pois ao somar dois n umeros s o aparece no visor o algarismo das unidades do resultado. Por exemplo, quando somo 6 e 7 aparece 3 no visor. Constru uma lista com 2005 algarismos 8, 6, 4, 0, 4, 4, 8, 2, 0, ... da seguinte forma: o terceiro algarismo da lista e os seguintes s ao o resultado da soma dos dois algarismos anteriores, como aparece no visor da minha calculadora. Qual eou ltimo algarismo dessa lista? 8. Tenho o triplo da idade que voc e tinhas quando eu tinha a idade que voc e tem. Quando voc e tiver a idade que eu tenho, teremos juntos 56 anos. Qual e a minha idade atual? 9. Esmeralda, de olhos vendados, retira cart oes de uma urna contendo inicialmente 100 cart oes numerados de 1 a 100, cada um com um n umero diferente. Qual e o n umero m nimo de cart oes que Esmeralda deve retirar para ter certeza de que o n umero do cart ao seja um m ultiplo de 4? 10. A m ae de Ana Margarida vende doces e pediu-lhe que embrulhasse 2003 bombons de 5 cores diferentes em pacotes com 3 de forma que em cada pacote os bombons fossem da mesma cor. Como recompensa prometeu-lhe que poderia comer os que restassem quando j a n ao fosse poss vel fazer mais embrulhos. Sabendo que dos 2003 bombons, 388 eram brancos, 396 amarelos, 406 verdes, 405 vermelhos e 408 castanhos, quantos bombons p ode a Ana Margarida comer e de que cor eram? 11. Considere uma sequ encia (1, 2, 2, 3, 3, 3, 4, 4, 4, 4, 5, 5, 5, 5, 5, ...) cujos termos s ao os inteiros consecutivos em ordem crescente, e na qual o inteiro n ocorre n vezes. Qual o resto da divis ao por 5 do 1993o termo desta sequ encia? 12. Determine o lado do quadrado inscrito em um tri angulo equil atero

Olimp ada de Matem atica do Estado de Goi as

(o quadrado ter a dois v ertices sobre um lado do tri angulo e os outros dois v ertices nos outros lados do tri angulo).

13. Dez n umeros inteiros (n ao necessariamente distintos) s ao tais que, excetuando-se um deles, as somas de todos os outros nove (dependendo daquele que e omitido) s ao iguais a 90, 91, 92, 93, 94, 95, 96, 97 e 98. Determine esses n umeros. 14. Prove que o produto das 99 fra c oes da forma 2 e maior do que 3 .
k 3 1 k 3 +1 ,

k=2, 3, ..., 100

15. Na gura o tri angulo ABC e is osceles, B AC = 20 e BC = BD = BE . Determine a medida do angulo B DE .

16. Sejam a e b n umeros reais tais que a2 = 6b + 5ab e b2 = 6a + 5ab. Determine o valor de ab. 17. O desenho ` a direita representa dois quadrados menores congruentes de lado 20 e um quadrado maior. O v ertice O eou nico ponto comum aos dois quadrados menores e e o centro do quadrado maior. Os v ertices A, O e B est ao alinhados e a area da regi ao do quadrado maior n ao pintada e igual a 36% da area de toda a regi ao pintada. Qual ea area do quadrado maior?

Olimp ada de Matem atica do Estado de Goi as

18. Em uma ilha deserta havia tr es homens e um macaco. Durante o dia os homens colheram cocos e deixaram a partilha para o dia seguinte. Durante a noite, um dos homens acordou e resolveu pegar a sua parte. Dividiu a pilha de cocos em tr es partes iguais, observou que sobrava um coco, deu este coco para o macaco, retirou e guardou a sua parte. Mais tarde, o segundo homem acordou e fez a mesma coisa que o primeiro, dando tamb em um coco para o macaco. Uma hora depois, o terceiro homem acordou e repetiu o que os outros dois haviam feito, dando um coco para o macaco. Na manh a seguinte os tr es homens, sem contar uns aos outros que j a tinham pego sua parte, repartiram os cocos que restavam em partes iguais, observaram que sobrou um coco e deram-no para o macaco. Qual e o menor n umero de cocos que a pilha inicial poderia ter? 19. Distribuir os n umeros 0, 1, 2, ..., 9 pelos c rculos da gura abaixo de forma que a soma dos n umeros colocados nos v ertices de cada tri angulo sombreado seja a mesma para todos os tri angulos. Descubra quantas solu c oes diferentes existem (n ao vale rodar o tri angulo ou fazer simetrias).

20. A equa c ao x3 + px + q = 0 tem tr es ra zes reais distintas. Prove que p < 0. 21. Num tri angulo ABC , seja E o p e da altura desde A sobre BC . , onde r e o raio da circunfer encia circunscrita, Demonstrar que AE = b.c 2r b = AC e c = AB .

Olimp ada de Matem atica do Estado de Goi as

22. Seja n inteiro positivo. De quantas maneiras podemos distribuir n + 1 brinquedos distintos para n crian cas de modo que toda crian ca receba pelo menos um brinquedo? 23. A circunfer encia inscrita no tri angulo ABC tem o centro O e e tangente aos lados BC , AC e AB nos pontos X , Y e Z , respectivamente. As retas BO e CO intersectam a reta Y Z nos pontos P e Q respectivamente. Demonstrar que se os segmentos XP e XQ t em o mesmo comprimento, ent ao o tri angulo ABC e is osceles. 24. A sequ encia Fn e denida por F1 = F2 = 1 e Fn = Fn1 + Fn2 para n 3. Encontre todos os pares de inteiros positivos (m, n) tais que Fm .Fn = mn. 25. Na gura W S e HI s ao cordas que se intersectam no ponto G, e RT e bissetriz do angulo W GI . Prove que W R T S = RI HT .

26. Mostre que o per metro de um pol gono regular de n lados inscrito em um c rculo de raio R e Pn = 2nR sen( 180 n ). 27. Na gura, ABCDE e um pent agono regular e AEF e um tri angulo equil atero. Seja P um ponto sobre o segmento BF , no interior de ABCDE , e tal que o angulo P EA mede 12o , como mostra a gura abaixo. Calcule a medida em graus, do angulo P AC .

Olimp ada de Matem atica do Estado de Goi as

28. Determine todas as solu c oes da equa c ao n2n1 + 1 = m2 , com n e m naturais. 29. S ao dadas duas circunfer encias secantes, com pontos de intersec c ao C e D . Tra ca-se por C uma secante ` as duas circunfer encias, que intercepta uma delas em E e a outra em F . Mostre que o angulo E DF e constante.

30. Uma seq u encia e denida por a1 = 8, a2 = 18 e an+2 = an+1 an para todo natural n 1. Determine todos os valores de n para os quais an e quadrado perfeito.

Revista da Olimp ada - IME - UFG, no - 7, Setembro 2008,

8-19

Solu c oes da Colet anea de Problemas

1. a) Sejam a, b, c, d, nesta ordem, os n umeros escolhidos para os v ertices de uma das faces. As arestas desta face ter ao n umeros iguais a: a + b, b + c, c + d, d + a, e o n umero da face ser a a + b + c + d + (a + b) + (b + c) + (c + d) + (d + a) = 3a + 3b + 3c + 3d = 3(a + b + c + d). Logo, o valor das faces e m ultiplo de tr es, e como 100 n ao e m ultiplo de tr es, nenhuma face pode ter esse valor. b) Sejam a, b, c, d, e, f, g, h os n umeros escolhidos para os v ertices. A soma dos 26 n umeros e igual a; a+b+c+d+e+f +g+h +
v ertices

+ (a + b) + (b + c) + (c + d) + (d + a) + (a + e) + (b + f )+ +(d + h) + (c + g) + (e + f ) + (e + h) + (h + g) + (g + f )+
arestas

+ 3(a + b + c + d) + 3(a + e + f + b) + 3(e + f + g + h)+ +3(h + g + c + d) + 3(a + e + d + h) + 3(b + f + c + g),


faces

que e igual a 13(a + b + c + d + e + f + g + h). Logo, a soma e m ultiplo de 13, e como 1995 n ao e m ultiplo de 13, n ao e poss vel fazer tal escolha. 2. Sejam, A o homem que possu a 5 p aes e 5 litros de agua, e B o homem que possu a 3 p aes e 3 litros de agua. Somando o que cada homem possu a, temos 8 p aes e 8 litros de agua. Cada um dos tr es homens comeu e bebeu quantidades iguais de p aes 8 de p a es e bebeu e agua, respectivamente, logo cada homem comeu 8 3 3 litros de agua. 8 7 Ent ao o homem A doou ao homem que nada tinha 5 3 =3 de p aes 7 8 1 1 e 3 litros de agua, e o homem B doou 3 3 = 3 de p ao e 3 de litro de

Olimp ada de Matem atica do Estado de Goi as

gua. Portanto, para a recompensa ser distribu a da de maneira justa, o homem A deve receber 7 moedas e o homem B, 1 moeda. 3. Se umas das bolas n ao fosse branca, as bolas deveriam ser de duas das tr es cores: verde, amarelo ou preto. Mas isso e absurdo, pois ter amos dois palpites completamente errados: o azul e vermelho e o branco e uma das cores verde, amarelo ou preto. Logo o branco e uma das cores e obviamente o amarelo, verde e o preto n ao s ao uma das cores, pois ningu em adivinhou as cores. O vermelho tamb em n ao e uma das cores, pois do contr ario n ao ter amos palpites completamente errado. Logo o palpite completamente errado e amarelo e vermelho e as cores certas s ao branco e azul. 4. Usando a regra dada na quest ao, podemos preencher as linhas da seguinte forma,

Assim, 42 = 13 + x + 11 + 2x = 24 + 3x. Logo, x = 6. 5. Seja y , o n umero no canto superior direito do quadrado m agico. Temos, pela regra do quadrado m agico, que a soma ao longo da diagonal que cont em y deve ser a mesma que a soma da coluna que o cont em, ou seja, 26 + 14 + y = y + x + 13 26 + 14 = x + 13 x = 27 . 6. Em lia escreveu em cada uma das primeiras linhas, na seguinte ordem, 01, 1011, 1031, 102113, 10311213, 10411223, 1031221314, 1041222314, 1031321324, 1031223314, 1031223314, ..., e percebeu que, a partir da 10a - linha, o n umero 1031223314 come ca a repetir. Portanto os dois primeiros algarismos da esquerda do n umero que a ela digitou na 2006 - linha ser ao 1 e 0.

Olimp ada de Matem atica do Estado de Goi as

10

7. Sabendo que a primeira parte da seq u encia e 8, 6, 4, 0, 4, 4, 8, 2, 0, se continuarmos a seq u encia teremos 8, 6, 4, 0, 4, 4, 8, 2, 0, 2, 2, 4, 6, 0, 6, 6, 2, 8, 0, 8, 8, 6, 4, ..., logo podemos perceber que a partir do 20o - algarismo a seq u encia come ca a repetir. Como 2005 = 100 x 20 + 5, e s o analisarmos o 5o - n umero da seq u encia, assim o u ltimo algarismo e o n umero 4. 8. Tu tinhas uma idade que chamaremos de x e hoje tens uma idade que chamaremos de y . Eu tenho o triplo da idade que tu tinhas quando eu tinha a tua idade atual y , ou seja eu tenho 3x. Eu tinha y e agora tenho 3x, portanto temos y x = 3x y 4x = 2y x = 1 y. 2

Ent ao, substituindo o valor de x, temos: tu tinhas 1 2 y e agora tens 3 y. y . Eu tinha y e agora e agora tenho 2 Olhando para a segunda frase: quando tu tiveres a idade que eu tenho, teremos juntos 56 anos. 1 Tu tens y e para ter minha idade deve-se somar ` a tua idade 2 y , assim 1 3 a tua idade devemos somar ` a minha voc e ter a 2 y . Como somamos 2 y ` tamb em, assim eu terei 2y , da temos: 7 3 y + 2y = 56 y = 56 y = 16. 2 2 3 Logo, a minha idade e 16 = 24 anos. 2 9. Como de 1 a 100 existem 25 m ultiplos de 4, 75 n umeros n ao s ao m ultiplos de 4, como Esmeralda corre o risco de tirar esses 75 n umeros respectivamente, ela deve tirar no m nimo 76 n umeros para garantir que um deles e m ultiplo de 4. 10. Sabemos que dos 2003 bombons, 388 eram brancos, 396 amarelos, 405 vermelhos, 406 verdes e 408 castanhos. Como 396, 405 e 408 s ao m ultiplos de tr es, n ao sobraram bombons amarelos, vermelhos e castanhos. Mas, como 388 = 129 x 3 + 1, 406 = 135 x 3 + 1, sobrou-lhe um bombom branco e um verde, Ent ao Ana Margarida comeu dois bombons, um branco e um verde.

Olimp ada de Matem atica do Estado de Goi as

11

11. Observe que at eou ltimo 1 escrevemos 1 n umero. At eou ltimo 2 escrevemos 1 + 2 n umeros. At eou ltimo 3 escrevemos 1 + 2 + 3 n umeros. At eou ltimo 4 escrevemos 1 + 2 + 3 + 4 n umeros, etc. Como a seq u encia foi constru da respeitando esta condi c ao, temos que at e ou ltimo n umero do tipo k teremos escrito 1 + 2 + 3 + + k k )k (soma n umeros. Mas essa soma 1 + 2 + 3 + + k e igual a (1+ 2 dos primeiros termos de uma progress ao aritm etica,ver [6], p ag. 51-60 e 61-70) ent ao fa camos essa soma igual a 1993. k )k Se (1+ = 1993, teremos k2 + k 3986 = 0, que, resolvendo, nos 2 d ak ao conv em) e k e estranho, = 63, 6 (n = 62, 6. Mas esse resultado j a que k deve ser um n umero inteiro. O que aconteceu? Certamente o 1993o n umero escrito n ao eou ltimo de uma seq u encia de n umeros iguais. .62 = 1953, que signica Vejamos se k for igual a 62, teremos (1+62) 2 que o 1953o - n umero desta seq u encia e o u ltimo 62 escrito. Logo, o 1993o - n umero e um dos n umeros 63 que ser ao colocados na seq u encia, como 63 = 12 5 + 3, o resto da divis ao por 5 e 3. 12. Sejam, a o lado do tri angulo equil atero, l o lado do quadrado, e a 3 h = 2 a altura do tri angulo equil atero.

C = ABC = 60o e Temos que M N e paralelo a AB , logo M N o N M C = B AC = 60 , ent ao M N C e equil atero de lado l. Como os tri angulos M N C e ABC s ao semelhantes, temos: hl ah l = lh = ah la l = . a h a+h Logo, a 3 a2 3 . l= 2a + a 3 2 + 3

Olimp ada de Matem atica do Estado de Goi as

12

13. Sejam n1 n2 n10 os dez inteiros e seja S = n1 + n2 + + n10 a sua soma. Excetuando-se n1 , a soma resultante e S n1 ; da mesma forma, se n2 for excetuado, o resultado e S n2 e assim sucessivamente. Se os 10 forem adicionados, obtemos (S n1 ) + (S n2 ) + + (S n10 ) = 10S S = 9S. Se obtemos apenas 9 resultados distintos,conforme enunciado, alguma soma e repetida. Seja x esta soma repetida, ent ao a soma dos 10 resultados e: 90 + 91 + 92 + 93 + 94 + 95 + 96 + 97 + 98 + x Pelo argumento acima, esta soma e tamb em igual 9S ; da 846 + x = 9S , o que nos leva a x = 9(S 94) e isto nos mostra que x = 90 e S = 104. Subtraindo-se 90, 91, 92, , 98, com 90 ocorrendo duas vezes, obtemos os inteiros 6, 7, 8, 9, 10, 11, 12, 13, 14 e 14. 14. Seja P = (23 1).(33 1).(43 1) (1003 1) . (23 + 1).(33 + 1).(43 + 1) (1003 + 1)

Agora, um exame nada complicado nos conduz aos seguintes resultados: Para um dado k, o primeiro fator na fatora c ao de (k + 2)3 1 e igual ao 3 primeiro fator de k + 1, ou seja, k + 1. Al em disso, o segundo fator de k3 1, que e k2 + k + 1, e igual ao segundo fator de (k + 1)3 + 1. Logo 1 2 10101 2 10101 P = = 3 100 101 3 10100 Como
10101 10100

Uma vez que n3 1 = (n 1)(n2 + n + 1) e n3 + 1 = (n + 1)(n2 n + 1) temos: (1 7).(2 13).(3 21) (99 10101) . P = (3 3).(4 7).(5 13) (101 9901)

> 1, temos P > 2 3.

15. Como ABC e is osceles, temos que B = C , e que A + B + C = 180o o ent ao, 2B = 160 portanto B = 80o e C = 80o . No tri angulo, BC = BD ent ao C = C DB = 80o . Como a soma dos angulos internos de um tri angulo e 180o , temos que C BD = 20o .

Olimp ada de Matem atica do Estado de Goi as

13

Ora B = E BD + C BD , portanto 80o = 20o + E BD logo E BD = 60o . Sabemos que no tri angulo BED , BE = BD . Temos que o angulo o o B ED = B DE , logo 2B DE = 120 B DE = 60 . 16. Subtraindo as duas equa c oes dadas temos, a2 b2 = 6(b a), como a2 b2 = (a b)(a + b) temos (a b)(a + b + 6) = 0. Como a = b, temos a + b = 6. Elevando a + b = 6 ao quadrado a2 + b2 = 36 2ab. Agora, somando as equa c oes dadas no enunciado e usando o fato que a + b = 6 temos, a2 + b2 = 6(b + a)+ 10ab = 36+ 10ab. Portanto, 36+ 2ab + 10ab = 36, o que nos d a ab = 6. 17. Como o centro do quadrado maior est a no encontro dos v ertices dos quadrados menores, temos que os quadrados menores dividem o quadrado maior em quatro quadril ateros congruentes. Logo, a a rea pintada e igual a soma das areas dos quadrados menores, ou seja, 2 (20 20) = 800. Como a area pintada do quadrado maior e igual ` a sua area n ao pintada, conclu mos que a area total do quadrado maior e igual a 72% da area total pintada, ou seja, 0, 72 800 = 576. 18. Seja x a quantidade de cocos que eles colheram. 1 O 1o - homem: d a um coco para o macaco, pega x 3 cocos para ele, 2 sobrando assim 3 (x 1) cocos. O 2o - homem: d a um coco para o macaco, pega ele, e sobram 4 ( x 1) 2 9 3 cocos.
2 (x1)1 3

cocos para

O 3o - homem: d a um coco para o macaco, pega 9 3 3 cocos para 8 ele e sobram 27 (x 1) 10 9 cocos. 8 De manh a h a ainda 27 (x 1) 10 9 cocos. Dando um coco para o 10 8 umero deve ser m ultiplo macaco sobram 27 (x 1) 9 1 cocos e esse n 8 de tr es, ou seja, 27 (x 1) 10 1 = 3 y . Com isso temos a seguinte 9 equa c ao: 8x 81y = 65. Resolvendo essa equa c ao diofantina, ver [7] p ag. 110-118, segue que x = 81t 2 e y = 8t 1. O menor valor de t que torna y positivo e t = 1. Logo, para t = 1 temos x = 79. 19. Sejam A, B , C , D , E , F , G, H , I e J os n umeros que est ao nos c rculos, conforme a gura abaixo.

(x1) 5

Olimp ada de Matem atica do Estado de Goi as

14

Como A, B, . . . , J valem 0, 1, 2, . . . , 9, tem-se A + B + C + D + E + F + G + H + I + J = 45. Al em disso A+B+C =B+D+E =C +E+F =D+G+H = E +H +I = F +I +J = x A soma dos n umeros dos tr es tri angulos que n ao cont em E vale 3x = 45 E . Da E s o pode ser 0, 3, 6 ou 9, caso contr ario x n ao seria inteiro. Nota-se ainda que E est a em tr es tri angulos, logo devem existir B, C, D, F, H, e I tais que B + D = C + F = H + I = x E . E = 0 x = 15 e x E = 15 s o pode ser escrito como 7 + 8 ou 9 + 6, mas s ao necess arias 3 somas, logo n ao pode ser 0. E = 3 x = 14 e x E = 11 pode ser escrito como 5 + 6 ou 7 + 4 ou 2 + 9. Ent ao E = 3 e poss vel. E = 6 x = 13 e x E = 7 pode ser escrito como 0 + 7 ou 2 + 5 ou 4 + 3. Ent ao E = 6 e poss vel. E = 9 x = 12 e x E = 3 s o pode ser escrito como 0 + 3 ou 1 + 2. Logo E n ao pode ser 9. Assim temos duas solu c oes:

Caso E = 3 20. Sejam x1 , x2 e x3 as ra zes da equa c ao. Pelas Rela c oes de Girard temos: x1 + x2 + x3 = 0 e

Caso E = 6.

x1 x2 + x1 x3 + x2 x3 = p.

Olimp ada de Matem atica do Estado de Goi as

15

Como
2 2 (x1 + x2 + x3 )2 = x2 1 + x2 + x3 + 2(x1 x2 + x1 x3 + x2 x3 ), 2 2 0 = x2 1 + x2 + x3 + 2p p = 2 2 (x2 1 + x2 + x3 ) . 2

2 2 Como x1 , x2 e x3 s ao distintos ent ao x2 1 + x2 + x3 > 0 e p < 0.

21. Seja AD o di ametro que sai do ponto A. Como ADC e ABC subentendem o mesmo arco, temos que ADC = ABE . O tri angulo ADC e ret angulo em C pois AD e di ametro da circuno fer encia. Sendo AEB = 90 , temos que o tri angulo ABE e semelhante ao tri angulo ADC , ent ao AE c AE AB = = . AD AC 2r b Logo, AE = b.c . 2r

22. Observe que teremos 1 crian ca com 2 brinquedos, enquanto cada uma das n 1 crian cas restantes ter a apenas 1 brinquedo. Assim, temos n+1 n possibilidades para a escolha da felizarda crian ca, e possibi2 lidades para escolher os dois brinquedos desta crian ca. Restando n 1 brinquedos e n 1 crian cas, temos (n 1)! modos de distribuir estes brinquedos entre estas crian cas. Assim, temos um total de n n+1 n+1 (n 1)! = n! 2 2

modos de distribuir os n + 1 brinquedos entre as n crian cas. 23. Temos que XP e XQ t em o mesmo comprimento, logo subentendem arcos congruentes, da temos que P OX = QOX . Como a circunfer encia e tangente ao tri angulo ABC , temos que OZ AB , OX BC e OY AC . Assim, temos que X BZ = 180o e que X CY = 180o , ent ao X BZ = X CY , onde = P OX . Logo, o tri angulo e is osceles. 24. Os primeiros valores da seq u encia s ao: F1 = 1, F2 = 1, F3 = 2, F4 = 3, F5 = 5, F6 = 8, F7 = 13, F8 = 21, F9 = 34.

Olimp ada de Matem atica do Estado de Goi as

16

Nota-se que, para n > 7, Fn > 2n. De fato, por indu c ao, se Fn > 2n e Fn+1 > 2(n + 1) ent ao Fn+2 = Fn+1 + Fn > 2(n + 1) + 2n > 2(n + 2). Portanto Fn > 2n > n para n > 7, de modo que para resolver as equa c oes Fn = n, basta testar os valores de n menores ou iguais a 7. Se n > 5, de Fm Fn = mn devemos ter Fm < m, onde m < 5. Logo pelo menos um dos n umeros m e n deve ser no m aximo 5. Suponha, sem perda de generalidade, n 5. Observando os poss veis valores de n: n = 1 Fm = m, cujas solu c oes s ao m = 1 e m = 5. n = 2 Fm = 2m, que n ao possui solu c ao. n = 3 2Fm = 3m, que n ao possui solu c ao. n = 4 3Fm = 4m, que possui uma u nica solu c ao m = 6. n = 5 Fm = m, cujas solu c oes s ao m = 1 e m = 5. Assim os pares (m, n) que satisfazem a rela c ao pedida s ao: (1, 1), (1, 5), (4, 6), (5, 1), (5, 5), (6, 4). 25. Sabemos que I GR = W GR e como H GT e S GT s ao opostos pelo v ertice a I GR e W GR, logo I GR = W GR = H GT = S GT. Como os angulos W IH e W SH subentendem o mesmo arcoW H , eles s ao congruentes, analogamente provamos que S HI = S W I . Como a soma dos angulos internos de um tri angulo vale 180o temos que S T G = T RG e que H T G = W RG. Assim, W RG = HT G e IGR = ST G. Portanto, WR RI = W R T S = RI HT . HT TS 26. Seja AC um dos lados do pol gono e o angulo AOC , onde O e 360o o centro da circunfer encia. Temos que = n . Seja OB a bissetriz

Olimp ada de Matem atica do Estado de Goi as


o

17

180 angulo AOC do angulo , logo, 2 = n . Como AC = 2AB , pois o tri e is osceles e usando as rela c oes trigonom etricas no tri angulo ret angulo AOB temos,

sen (

180o 180o AB )= AB = R sen ( ). n R n


o

Ent ao AC = 2R sen ( 180 metro do pol gono a soma de n ), e como o per todos os lados, Pn = nAC Pn = 2nR sen ( 180o ). n

27. Primeiro observamos que os angulos internos de um pent agono re.180o o . O tri = 108 a ngulo AEF e equil a tero, logo, gular medem (52) 5 o o A = E = F = 60 . Como B AF = B AE + E AF, B AF = 168 e como o tri angulo BAF e is osceles, ABF = AF B = 6o . No tri angulo P EF , temos E F P = 60o 6o = 54o , P EF = 60o + 12o = 72o e

F P E = 180o 72o 54o = 54o . Logo o tri angulo P EF e is osceles, com P E = F E . Ent ao o tri angulo o 12o = AP E tamb em e is osceles com AE = P E e P AE = AP E = 180 2 180o 108o o o 84 . O tri angulo CAB e is osceles, da C AB = = 36 , al em 2 o o o disso C AE = 108 36 = 72 . Portanto, P AC = P AE C AE = 84o 72o = 12o . 28. A equa c ao e equivalente a n2n1 = (m + 1)(m 1). Observe que se n = 1, 2, 3, ou 4 temos m2 = 2, 5, 13, ou 33, respectivamente, o que implica que m n ao e um n umero inteiro. Podemos supor ent ao que

Olimp ada de Matem atica do Estado de Goi as

18

n > 4. Como m2 e mpar, temos m mpar, digamos m = 2l + 1. A equa c ao ca ent ao n2n3 = l(l + 1). Portanto, 2n3 divide l ou (l + 1). Assim, l + 1 2n3 e l n, onde n + 1 2n3 . Mostraremos, por indu c ao, ver [6] p ag.71-78, que n + 1 < 2n3 para n > 5. Para n = 6 (base de indu c ao), temos 6 + 1 = 7 e 263 = 8. Supondo que a desigualdade e v alida para n = k, provemos que a mesma e v alida para n = k + 1 (passo indutivo). De fato, por hip otese de indu c ao temos k + 1 < 2k3 . Como k + 2 < k 2 2(k + 1), temos k + 2 < 2 , completando a demonstra c ao. Assim, basta testar n = 0 e n = 5. Portanto as solu c oes s ao (m, n) = (1, 0) e (m, n) = (9, 5). 29. Trace por C uma secante as duas circunfer encias que ` as interceptam em G e H . Temos que F + E + E DF = 180o e H + G + H DG = 180o . O angulo E F D subtende o arco CD , e o angulo GHD tamb em subtende o arco CD, logo E F D = GHD . Analogamente provamos que F ED = H GD . Assim, temos que E DF = 180o E F D F ED e GDH = 180o E F D F ED . Logo, E DF = GDH para qualquer reta que passe por C e seja secante ` as duas circunfer encias. 30. Observamos que a1 = 23 , a2 = 32 .2, a3 = 24 .32 = (22 .3)2 , a4 = 25 .34 , a5 = 29 .36 , a6 = 214 .310 = (27 .35 )2 , a7 = 223 .316 . Assim, an e quadrado perfeito quando os expoentes do 2 e do 3 forem pares. Os expoentes do 3 s ao sempre pares por serem uma soma de dois pares. Os expoentes do 2 formam uma seq u encia assim denida: b1 = 3 b =1 2 bn+1 = bn + bn1 , n 2. b1 1, b2 1, b3 0, b4 1, b5 1, b6 0, Como a seq u encia e determinada por dois valores consecutivos bn e bn+1 ,

Considerando bn m odulo 2:

Olimp ada de Matem atica do Estado de Goi as

19

temos que ela e peri odica m odulo 2, com per odo 3. Assim an e quadrado perfeito se, e somente se, n e m ultiplo de 3.

Bibliograa
[1 ] Barbosa, J. L. M., Geometria Euclidiana Plana, Cole c ao do Professor de Matem atica, Rio de Janeiro, SBM, 2004. [2 ] Moreira, C. ; Motta, E.; Tengan, E.; Am ancio, L.; Saldanha, N.; Rodrigues, P.; Olimp adas Brasileiras de Matem atica- 9a -a a 16 - ,SBM, 2003. [3 ] Olimp ada Iberoamericana de Matem atica e Olimp ada Internacional de Matem atica, em www.obm.org.br/provas.htm#imo [4 ] Olimp adas Portuguesas de Matem atica, www.mat.uc.pt/opm [5 ] Revista do Professor de Matem atica, no - 16, S ao Paulo, SBM, 1990. [6 ] Revista da Olimp ada de Matem atica do Estado de Goi as, no -2e o n - 3, 2001/2. [7 ] Santos, A. L.; Wagner, E.; Agostino, R. F. W.; Olimp ada de matem atica do Estado do Rio de Janeiro, S ao Paulo: Atual; Rio de Janeiro : SBM, 1996. Edm eia Fernandes da Silva Endere co: Universidade Federal de Goi as Instituto de Matem atica e Estat stica Caixa Postal 131 74001-970 - Goi ania - GO - Brasil edmeia@mat.ufg.br

Revista da Olimp ada - IME - UFG, no - 7, Setembro 2008,

20-23

Classicados na XIV OMEG - 2005

N vel 1 (5a - e 6a - S eries do Ensino Fundamental)


Primeiro Lugar Marcelo Abdala Daher /Col egio Crescer - An apolis. Segundo Lugar J ulio Ferreira Soares Filho /Col egio Galileu - An apolis. Terceiro Lugar Iuri Rezende Souza /Ed. Nascentes do Araguaia - Mineiros. Franco Puglisi /Col egio Galileu - An apolis. Andr e Santiago Beires / Col egio Crescer - An apolis. o Honrosa Menc a Danilo Rebou cas Fernandes de Lima /Col. Agostiniano - Goi ania. Fernanda Lu za Hor acio Buta /Col egio Crescer - An apolis. Mateus Miranda Andrade /Col egio Crescer - An apolis. Mariana Gomes Gerais /Col egio Crescer - An apolis. Jhonantans Moraes Rocha /Col egio Intera c ao - Ap. de Goi ania. Douglas Flor encio de Sousa /Col egio Ateneu Dom Bosco - Goi ania. Igor Muniz Soares /Col egio Expovest - Goi ania Lucas Moraes Silva /Col egio Expovest - Goi ania Pedro Maur cio de Sousa /Col egio Crescer - An apolis.

Olimp ada de Matem atica do Estado de Goi as

21

N vel 2 (7a - e 8a - S eries do Ensino Fundamental)


Primeiro Lugar Pedro Victor Aniz Gomes de Oliveira /Inst. Maria Auxiliadora Goi ania Segundo Lugar Sara de Oliveira Santos /Inst. Presbiteriano de Educa c ao - Goi ania Terceiro Lugar Nicolas Cruvinel Lindo /Col egio Santo Agostinho - Goi ania Menc ao Honrosa Alice Duarte Scarpa /Col egio Pr e-medico - Goi ania Mariana Rodrigues Alves /Inst. Presbiteriano de Educa c ao - Goi ania. Pedro Arthur F. Borges /Col egio Integrado Ja o - Goi ania. Murilo Antunes de Castro /Col egio Disciplina - Goi ania. Filipe R. Parrode /Col egio Vict oria Figueiredo - Goi ania. David Issa Mattos /Col egio Crescer - An apolis X enia Larissa Motta Fernandes /Inst. Ma - Auxiliadora - Goi ania. Andr e Cardoso Bernardes /Col egio Disciplina - Goi ania. Amanda Barroso de Freitas /Inst. Pres. de Educa c ao - Goi ania. Marina Berqu o Peleja /Inst. Presbiteriano de Educa c ao - Goi ania. Juliana de Faria Silva /Col egio Ateneu Dom Bosco - Goi ania. Ian Castilho Caldeira Brant /Col egio Santo Agostinho - Goi ania. Du lio Cardoso de S a /Col. E. Dom Pedro I - Aparecida de Goi ania. Arthur Magalh aes de Oliveira /Inst. Pres. de Educa c ao - Goi ania. Bruno Ara ujo Fran ca /Col egio Marista - Goi ania. Laura Izidoro Salerno /N ucleo Educativo - Catal ao. Marcos Celestino Carvalho J unior /Col egio Goyases - Goi ania. Bruno Vargas Adorno /Centro Ed. Sesc Cidadania - Goi ania. Jo ao Victor Batista Gordo /Col egio Agostiniano - Goi ania.

Olimp ada de Matem atica do Estado de Goi as

22

Lara Cristina Duarte /N ucleo Educativo - Catal ao. Davi de Castro Silva /Col egio Marista - Goi ania. Danilo Sulino S. Pinto /Col egio Discplina - Goi ania. H ericka de Paula Bento /Col egio Galileu - An apolis. Luciano Gomes de Sousa Filho /Col. Mons. Angelino - Inhumas. Paulo L ucio Bahia Silva J unior /Col egio Agostiniano - Goi ania. Hadassa Caruirane Bonif acio de Lima /Col egio Degraus - Goi ania. Let cia Cestari H. da Silva Campos /Col egio Agostiniano -Goi ania. Pedro Bittencourt Arruda /Col egio Marista - Goi ania. Rebeca Daher Fran ca /Col egio Crescer - An apolis. Vaniele Guimar aes carvalho /Ed. Nascente do Araguaia - Mineiros. Larissa Alves Carri ao /Col egio Disciplina - Goi ania. Luan Felipe R. Costa /Col egio Agostiniano - Goi ania. Lorrany Miquelante Yoshida /Col egio Marista - Goi ania. Leandro Sobocinski Castro /Col egio Integrado Ja o-Goi ania. Patr cia Ferreira de Freitas /Col. E. Divino Pai Eterno - Trindade. Matheus Henrique de A. Ara ujo /Inst. Pres. de Educa c ao - Goi ania. Hugo Brand ao Cardozo /Centro Ed. Sesc Cidadania - Goi ania.

N vel 3 (Ensino M edio)


Primeiro Lugar Pedro Henrique Guedes de Oliveira /Col egio Prevest - Goi ania. Segundo Lugar Vicente de Souza Cardoso J unior /Col egio Vis ao - Goi ania. Terceiro Lugar Luciana M. R. Salgado /Col egio Vis ao - Goi ania.

Olimp ada de Matem atica do Estado de Goi as

23

Erick Assis Lima /Col egio WR - Goi ania. o Honrosa Menc a Francisco Habib Issa Mattos /Col egio Galileu - An apolis. Leonardo Augusto de Oliveira /Col egio Vis ao - Goi ania. Filipe Louly Quinan Junqueira /Col egio WR - Goi ania. Iran Carlos M. F. Filho /Col egio WR - Goi ania. Guilherme Victor Humberto Soares Carneiro /Col. Agostiniano Goi ania. Rafael Soares da Mota /Col egio Uniclass Objetivo - Goi ania. Gustavo Medeiros da Silveira /Col egio WR - Goi ania. Lu z Miguel Perandini Barini /Col egio Expovest - Goi ania. Ricardo Nunes Martins /Col egio Vis ao - Goi ania. Ricardo Elias Ramos /Col egio Discplina - Goi ania. Amanda Guedes de Oliveira /Col egio Prevest - Goi ania. Vantuir santos j unior /Neo Instituto de Ensino - Porangatu. Paulo Henrique Bento Ribeiro /Col egio Discplina - Goi ania. Daniel Adam Miranda /Col egio Vis ao - Goi ania. Pedro Henrique Terra Estrela /Col egio WR - Goi ania.

Revista da Olimp ada - IME - UFG, no - 7, Setembro 2008,

24-27

Classicados na XV OMEG - 2006

N vel 1 (5a - e 6a - S eries do Ensino Fundamental)


Primeiro Lugar Mateus Mota Moraes /Instituto Pres. de Educa c ao - Goi ania. Segundo Lugar Matheus Felter Rocha /Escola Goi as - An apolis. Terceiro Lugar Pedro Augusto Machado Jorge /Centro Educacional - Goi ania. Ricardo Skrebsky Rubenich /Escola Professora Yolanda - Goi ania. o Honrosa Menc a Nath alia Gomes Mialichi /Col egio Disciplina - Goi ania. Vict oria Franco Gon calves /Escola Interam erica - Goi ania. Caio S. Carvalho /Escola Interam erica - Goi ania. Elias Achkar Filho /Col egio Crescer - An apolis. Giovanna Silva Bianchi /Col. Agostiniano N. S. de F atima - Goi ania. Guilherme Miranda Andrade /Col egio Galileu - An apolis. Lucas Cardoso de Oliveira /Inst. Pres. de Educa c ao - Goi ania. Lanna Franco Ribeiro /Escola Interam erica - Goi ania. Mateus Fonseca Mazzo /Col egio Crescer - An apolis. Victor Falc ao Pereira Costa /Col. Agostiniano N. S. de F atima Goi ania.

Olimp ada de Matem atica do Estado de Goi as

25 Agostiniano N. S. de

Victor Queiroz Santana Bernardes /Col. F atima - Goi ania.

Pl nio Marques Siqueira /Escola Interam erica - Goi ania. Allison Lindsay Marques da Silva /Inst. Pres. de Educa c ao Goi ania. Ernesto Leite Xavier Neto /Col egio Diocesano - Itumbiara. Nat alia Rodrigues Parrode /Col. Vict oria Figueiredo - Goi ania. Pedro Henrique Pereira Rocha /Col egio Galileu - An apolis. Wesley Fl avio de Lima J unior /Col egio Delta - An apolis.

N vel 2 (7a - e 8a - S eries do Ensino Fundamental)


Primeiro Lugar Felipe Rodrigues Parrode /Col egio Vict oria Figueiredo - Goi ania. Segundo Lugar J ulio Ferreira Soares Filho /Col. Santo Agostinho - Ap. de Goi ania. Terceiro Lugar Guilherme Rezende Barros /Col. Agostiniano N. S. de F atima Goi ania. o Honrosa Menc a Sara de Oliveira Santos /Inst. Pres. de Educa c ao - Goi ania. Marina Berqu o Peleja /Instituto Pres. de Educa c ao - Goi ania. Matheus de Oliveira Afonso Ogawa /Col. U. Objetivo - Goi ania. Andr e Santiago Beires /Col egio Crescer - An apolis. Lu a Alves Caetano de Oliveira /Col. Ateneu S. D. Bosco - Goi ania. Roberto Sobocinski Castro /Col egio Integrado Ja o - Goi ania. Arthur Magalh aes de Oliveira /Inst. Pres. de Educa c ao - Goi ania. Cleuter Ant onio Pigorelli Carneiro /Col egio Disciplina - Goi ania.

Olimp ada de Matem atica do Estado de Goi as

26

Fernando Henrique Coimbra Afonso /Inst. Pres. de Educa c ao Goi ania. Marco T ulio Jos e de Oliveira /Centro E. Sesc Cidadania - Goi ania. Ana Carina Peres Ferreira dos Santos /Col. Mons. Angelino Inhumas. Jeov a Barbosa Ramos Filho /Col egio E. Pe. N. Maranh ao Arzola Buriti Alegre. Laura de Freitas Severino /Col egio Vict oria Figueiredo - Goi ania. Ian Castilho Caldeira Brant /Prevest Unidade Sul - Goi ania. Iuri Rezende Souza /Educand ario N. do Araguaia - Mineiros. Yu Tzu Wu /Col egio Delta - An apolis. Matheus Fagundes de Azevedo /Col egio Crescer- Goi ania. Alexandre Lemes de Freitas /Col. Galileu Uni. Centro - An apolis. Vaniele Guimar aes Carvalho /Edu. Nascentes do Araguaia - Mineiros. Danilo Rebou cas /Col. Agostiniano N. S. de F atima - Goi ania. Tatiane Machado Ferreira /Col egio Exato - Ipor a. Luisa Maranh ao Silva /Col egio Delta - An apolis. Beatriz Ribeiro Kherlakian Barbosa /Col egio Profa - Yolanda - Goi ania. Bruno Franco Bel em /Col egio Vict oria Figueiredo - Goi ania. Carolina Guarni eri Gouveia /N ucleo Educativo Catal ao - Catal ao. Douglas Flor encio de Sousa /Col. Ateneu S. Dom Bosco - Goi ania. Jo ao Pedro Vivolo Lopes e Souza /Inst. Pres. de Educa c ao Goi ania. Joel Felippe Barbosa de Oliveira Velloso /Esc. Grandaso - Goi ania. Marcela Rodrigues de Magalh aes /Col. Agostiniano N. S. de F atima - Goi ania. Marcos Celestino Carvalho J unior /Col egio Goyases - Goi ania. Jean Carlos de Aguiar /Escola Rosa Tur stico Araujo - Anicuns.

Olimp ada de Matem atica do Estado de Goi as

27

N vel 3 (Ensino M edio)


Primeiro Lugar Guilherme Augusto Sousa Alc antara /Centro E. Paulo F. - Anglo Catal ao. Segundo Lugar Vantuir Santos J unior /Neo Instituto de Ensino - Porangatu. Terceiro Lugar Let cia Goulart Netto /Col egio Universit ario - Catal ao. Amanda Guedes de Oliveira /Col egio Prevest Centro - Goi ania. Let cia Maria Silva /Col. Estadual Jales Machado - Goian esia. o Honrosa Menc a C assio Sobocinski Castro /Col egio Integrado Ja o - Goi ania. Jos e Carlos Carvalho J unior /Col. E. Jos e L. de Almeida - An apolis.

Revista da Olimp ada - IME - UFG, no - 7, Setembro 2008,

28-33

Classicados na XVI OMEG - 2007

N vel 1 (6o - e 7o - Anos do Ensino Fundamental)


Primeiro Lugar Isabela Oliveira C. de Moura /Inst. Galileu de Ensino - An apolis. Segundo Lugar Lucas Carvalho Daher /Escola de E. Fund. Crescer - An apolis. Terceiro Lugar Roberta Sud aria Pinheiro /Col egio Marista - Goi ania. o Honrosa Menc a Let cia Duarte Rosique /Col. Vict oria Figueiredo - Goi ania. Nat alia Rodrigues Parrode /Col. Vict oria Figueiredo - Goi ania. Caio Ludovico Paranhos /Col. Goi ania. Agostiniano N. S. de F atima -

Pedro Augusto Machado Jorge /Centro Edu. Sesc Cidadania Goi ania. Yuri Luiz Dias Martins /Col egio Expressivo - Goi ania. Paula Santana Marra /Inst. Pres. de Educa c ao - Goi ania. Arthur Moraes do Lago /Col. Integrado Ja o - Goi ania. Gregory Augusto Curado Ribeiro /Centro Edu. OMNI - Goi ania. Gustavo Martins da Silva Gusm ao /Col egio Laruna - Goi ania. Jo ao Jorge Massaralla Neto /Col. Marista de Goi ania - Goi ania. Lucas Carvalhaes de Andrade /Esc. Monsenhor Angelino - Goi ania.

Olimp ada de Matem atica do Estado de Goi as

29

Ana Carolina M. Oliveira /Col. Diocesano de Itumbiara - Itumbiara. Felipe de Oliveira Emos /Externato S ao Jos e - Goi ania. Luisa Rezende Barros /Col. Agostiniano N. S. de F atima - Goi ania. Francine Carvalho Pietrobom /Col. Vict oria Figueiredo - Goi ania. Isabella Vecchi /Inst. Galileu de Ensino - An apolis. Lucas Guilherme Mendes Negreiro /Col. Santo Agostinho - Goi ania. Jo ao Victor Barbosa Paiva /Col. Marista de Goi ania - Goi ania. T ulio C esar de Brito Nunes /Col. Maria J ulia - Goi ania. Cassio Lazaro Narros Cabral /Edu. Nascentes do Araguaia - Jata . Gabrielle Macanham Guimar aes /Esc. An apolis. de E. Fund. Crescer -

Natielly Aleixo In acio /Inst. Pres. de Educa c ao - Goi ania. Felipe Fisher Mascarenhas /Col. Vict oria Figueiredo - Goi ania. Nath alia Laureano Prata Cardoso /Col. Batista Goiano - Goi ania. Ana Carolina Lopes Caixerta /Col. Uniclass/Objetivo - Goi ania. Igor Machado Peres /Col. Batista Goiano - Goi ania. Luiz Augusto Neto de Bastos /Inst. Galileu de Ensino - An apolis. Marina Miyuki Komatsu /Escola de E. Fund. Crescer - An apolis. N ataly Duarte Lopes da Costa /Escola Grandaso - Goi ania. Isabela Laguardia /Inst. Galileu de Ensino - An apolis. Lucas Freire Rassi /Inst. Pres. de Educa c ao - Goi ania. Marcos Mathias Pereira /Col. Agostiniano N. S. de F atima - Goi ania. St efano Maia Silva Cala ca /Col egio Lassale -Goi ania.

Olimp ada de Matem atica do Estado de Goi as

30

N vel 2 (8o - e 9o - Anos do Ensino Fundamental)


Primeiro Lugar Nat alia Ara ujo Ferreira /Col egio Integrado Ja o - Goi ania. Segundo Lugar Prata Iuri Rezende de Souza /Ed. Nascentes do Araguaia - Jata . Terceiro Lugar J ulio Ferreira Soares Filho /Col egio Santo Agostinho - Goi ania. o Honrosa Menc a Danilo Rebou cas Fernades de Lima /Col. Agostiniano N. S. de F atima - Goi ania. Pedro Henrique Pedrosa Torres /Col egio Marista de Goi ania - Goi ania. Roberto Sobocinski Castro /Col egio Integrado Ja o - Goi ania. Joel Felippe Barbosa de Oliveira Veloso /Escola Grandaso - Goi ania. Ygor Muniz Soares /Expovest - Goi ania. Bruna Bianco Hummel /Escola de E. Fund. Crescer - An apolis. Fernando Henrique Coimbra Afonso /Instituto Pres. de Educa c ao - Goi ania. Gabriel Borges /Col egio Integrado Ja o - Goi ania. Andr e Santiago Beires /Escola de E. Fund. Crescer - An apolis. Pedro Silva Lima /Col. Agostiniano N. S. de F atima - Goi ania. Ricardo Skrebsky Rubenich /Escola Profa - Yolanda - Goi ania. Paulo Henrique Pedroso de Lima /Col. Marista de Goi ania - Goi ania. Marcelo Abdala Daher /Escola de E. Fund. Crescer - An apolis. Alline Carvalho De Souza /Col. Prevest Unid. Sul - Goi ania. Douglas Flor encio de Sousa /Col. Salesiano Ateneu Dom Bosco Goi ania. Pedro Mauricio de Sousa /Escola de E. Fund. Crescer - An apolis. Jo ao Vitor Falchetti /Col. Rioverdense - Objetivo - Goi ania.

Olimp ada de Matem atica do Estado de Goi as

31

Thiago Oliveira Costa /Col. Prevest U. Sul - Goi ania. Isadora Carvalho Medeiros Francescantonio /Col. Integrado Ja oGoi ania. Marcela Cortes Ferreira /Inst. Galileu de Ensino - An apolis. Pedro Henrique Machado Godoy /Col. Vict oria Figueiredo - Goi ania. Franklin Roberto Dutra de Souza /Centro Edu. SESC Cidadania Goi ania. Roberto Carlos J unior /Col. Integrado Ja o - Goi ania. Jo ao Paulo Yoshio da Silva /Inst. Galileu de Ensino - An apolis. Sarah Gonsalves Cruz /Col. Prevest U. Sul - Goi ania. Thiago Nascimento Nogueira /Esc. Grandaso - Goi ania. Leticia Faria Bahia /Col. Batista Goiano - Goi ania. Philippe Borges Ruiz Lopes /Centro Ed. SESC Cidadania - Goi ania. Giovanna Silva Bianchi /Col. Agostiniano N. S. de F atima - Goi ania. Rafael Camargo Freitas /Col. Agostiniano N. S. de F atima - Goi ania. Arthur Henrique Camargo dos Santos /Col. Integrado Ja o - Goi ania.

N vel 3 (Ensino M edio)


Primeiro Lugar Rafael Elias Nascimento /Col egio Vis ao - Goi ania. Segundo Lugar Pedro Arthur Ferreira Borges /Col. Integrado Ja o - Goi ania. Terceiro Lugar Guilherme Augusto Sousa Alcantara /Col egio Universit ario - Catal ao. o Honrosa Menc a

Olimp ada de Matem atica do Estado de Goi as

32

Eduardo Rodrigues Silva Filho /Classe S. de Ensino - Goi ania. Fl avio Junio Rodrigues Mendes /Col egio Monsenhor - Goi ania. Andre Baramili Fleury Amorim /Col egio Avila - Goi ania. Renato Hugo Reis Borges /Col egio Vis ao - Goi ania. Igor Cardoso Amatte /Col egio Monsenhor - Goi ania. Jo ao Gabriel Moura Campos /Col. Integrado Ja o - Goi ania. C assio Sobocinski Castro /Col. Integrado Ja o - Goi ania. Jo ao Carlos Rocha de Borba /Col. Prevest U. Centro - Goi ania. Vin cius Mota Rezende /Col egio Integrado Ja o - Goi ania. Bruno C esar R. do Amaral /Inst. Galileu de Ensino - An apolis. Thiago Henrique Silva /Col. Prevest U. Centro - Goi ania. Fernado Teixeira Barros /Col egio Vis ao - Goi ania. Lorena de Sousa Brito /Col. Planeta Educacional - Goi ania. Andr e Lemes de Freitas /Inst. Galileu de Ensino - An apolis. Davi de Castro Silva /Col egio Vis ao - Goi ania. Augusto Ant onio Ribeiro Silva /Col egio Degraus - Goi ania. Pedro Henrique Peixoto Daco /Col. S ao F. de Assis - An apolis. Rafael Vinicius Britto Fernandes /Esc. Emp. Educacionais - Goi ania. F abio Umeno Ferreira /Col. Santo Agostinho - Goi ania. Humberto Furtado /Col. Agostiniano N. S. de F atima - Goi ania. Matheus Alves Farah /Col. Prevest U. Sul - Goi ania. Everton Lima Aleixo /Col. Planeta Educacional - Goi ania. Ian Castilho Caldeira Brant /Col. Prevest U. Centro - Goi ania. Let cia Goulart Netto /Col egio Universit ario - Catal ao. Henrique Coral de Azeredo /Col egio Vis ao - Goi ania. David Issa Mattos /Inst. Galileu de Ensino - An apolis. Douglas Souza Soares /Inst. Galileu de Ensino - An apolis.

Olimp ada de Matem atica do Estado de Goi as

33

Danilo Inacio de Souza Resende /Col egio Executivo - Goi ania. V tor Bueno Figueiredo de Paula /Col egio Einstein - Goi ania. Felipe de Andrade Neves /Col egio Vis ao - Goi ania. Fl avio Augusto Chavier Carneiro /Col. Prevest U. Centro - Goi ania. M arcio Andr e de Godoy Uema /Col egio Pardal - Goi ania.

Revista da Olimp ada - IME - UFG, no - 7, Setembro 2008,

34-35

Not cias
A IV Bienal da SBM realizar-se- a no per odo de 29/09 a 03/10/2008 no Dep. de Matem atica da Universidade Estadual de Maring a - UEM. http://www.dma.uem.br/bienalsbm. A XVII Olimp ada de Matem atica do Estado de Goi as ser a realizada em 27 de setembro de 2008 das 13:30h ` as 18:00h, nos campi da UFG de: Goi ania, An apolis, Catal ao, Ipor a, Itumbiara, Jata , Porangatu, Quirin opolis e Rialma. Para participar da XVII Olimp ada de Matem atica do Estado de Goi as a escola deve estar cadastrada. O cadastramento e as inscri c oes dever ao ser feitos diretamente pelo site www.mat.ufg.br at e 31 de agosto de 2008. Poder ao participar, por escola, at e: o o 10 estudantes no n vel 1 (6 - e 7 - ano do Ensino Fundamental); 10 estudantes no n vel 2 (8o - e 9o - ano do Ensino Fundamental); 10 estudantes no n vel 3 (Ensino M edio). A sele c ao dos estudantes para participarem da XVII OMEG car aa crit erio da escola, podendo ser utilizada a prova da 1a - fase da 29a - OBM e ou da 3a - OBMEP para esta sele c ao. A XXX Olimp ada Brasileira de Matem atica -OBM e realizada nos n veis 1, 2 e 3 em tr es fases: 1o - fase 9/06/2008 na escola. o 2 - fase 13/09/2008 na escola. 3o - fase 27 e 28/10/2008, no Instituto de Matem atica e Estat stica da UFG. Para participar da XXX OBM a escola dever a se cadastrar na Secretaria da OBM. A cha pode ser encontrada no site www.obm.org.br. A 4a - Olimp ada Brasileira de Matem atica das Escolas P ubicas -OBMEP est a sendo realizada em 2008 nos n veis 1, 2 e 3 em duas fases: 1a - fase 26/08/2008, na escola.

Olimp ada de Matem atica do Estado de Goi as

35

Mais de 18 milh oes de estudantes realizar ao as provas da 4a - OBMEP. a 2 - fase 8/11/2008. Resultado em Fevereiro de 2009. Mais informa c oes no site www.mat.ufg.br/obmep. Datas de Outras Olimp adas: A Olimp ada Brasileira de Matem atica - N vel Universit ario e realizada em duas fases. A primeira fase foi realizada dia 13 de setembro de 2008 e a segunda fase ser a nos dias 26 e 27 de outubro de 2008 no Instituto de Matem atica e Estat stica da UFG. Participam estudantes de v arios cursos universit arios [www.obm.org.br]. A Olimp ada de Maio e uma competi c ao realizada para jovens estudantes, disputada em dois n veis (N vel 1: para alunos at e 13 anos e N vel 2: para alunos de at e 15 anos), por pa ses da Am erica Latina, Espanha e Portugal. No Brasil a olimp ada de maio e aplicada apenas ` aqueles alunos premiados na Olimp ada Brasileira de Matem atica (medalhas de ouro, prata, bronze e men c oes honrosas) e na XVII Olimp ada de Maa tem atica do Estado de Goi as. A 14 - Olimp ada de Maio foi realizada no dia 12 de maio de 2008, nos locais designados por cada coordena c ao regional. Ser a realizado o XXIII Semana do IME no Campus Samambaia/ UFG, de 6 a 10 de outubro de 2008. www.mat.ufg.br.

Revista da Olimp ada - IME - UFG, no - 7, Setembro 2008,

36-48

Solu c oes Comentadas das Provas da XIV OMEG - 2005


Luciana Maria de Avila Rodrigues2 Resumo. Nesta se c ao apresentamos as solu c oes comentadas dos participantes da XIV OMEG.

N vel 1
Problema 1) Considere a express ao 3n + 7n N. Note que para n = 1 1 1 2005 tem-se 3 + 7 = 10. Decida se 3 + 72005 e ou n ao divis vel por dez. Justique sua resposta. Solu c ao apresentada por Andr e Santiago Beires e Mateus Miranda Andrade: O n umero 7 elevado a qualquer pot encia mpar, ter a como u ltimo n umero o 7 e o 3. Veja: 71 = 7; 73 = 343; 75 = 16807. O mesmo acontece com 3. Temos 31 = 3; 33 = 27; 35 = 243. Observe que os n umeros 3 e 7 aparecem alternados nas pot encias de 3n e 7n com mesmo expoente mpar. Quando fazemos a soma temos um n umero terminado em 0, logo divis vel por 10. Problema 2) H a um rel ogio cujo mostrador teve seus n umeros embaralhados. Ache todos os n umeros a partir das dez arma c oes seguintes. 1. O n umero na posi c ao F est a correto. 2. A diferen ca entre os n umeros nas posi c oes G e F e 6. 3. A soma dos n umeros A e G e treze.
GarAgradecemos o trabalho de digita c ao parcial do bolsista/PROEC/2005: Elis del da Costa Mesquita.
2

Olimp ada de Matem atica do Estado de Goi as

37

4. O n umero na posi c ao L e o dobro do n umero A, um ter co do n umero na posi c ao D e metade do n umero na posi c ao E. 5. Os n umeros D e J tamb em somam treze. 6. O n umero na posi c ao D e o qu adruplo de um dos n umeros adjacentes a ele. 7. O n umero na posi c ao D e o dobro do n umero na posi c ao H. 8. A soma dos seis primeiros n umeros (A a F ) e igual ` a soma dos seis u ltimos. 9. N ao h a nenhum par de n umeros consecutivos. 10. Nenhum n umero par est a entre dois n umeros mpares. Para a localiza c ao dos n umeros siga no mostrador abaixo, onde as letras de A a L foram utilizadas para facilitar o trabalho. Justique sua resposta. Solu c ao apresentada por Andr e Santiago Beires e Iuri Rezende Souza: Da arma c ao 1) temos que F = 5, de 2) temos G 5 = 6 G = 11; de 3) 11 + A = 13 A = 2; de 4) L = 2, D = 3L E = 2L L = 4, D = 12, E = 8; de 5) 12 + J = 13 J = 1. Da arma c ao 6) C = 3; da arma c ao 7) H = 6. Da arma c ao 10) segue que I = 10, pois restam os n umeros 7, 9 e 10. Da arma c ao 8) e do fato de que a soma dos n umeros de um rel ogio e 78, segue que A + B + C + D + E + F = 39 Da B = 9 e K = 7. Problema 3) Seja A um conjunto nito de n umeros racionais positivos com a seguinte propriedade: se q 2 3 p A ent ao A. Por exemplo, se A, ent ao tamb em A. q p 3 2 e G + H + I + J + K + L = 39.

Olimp ada de Matem atica do Estado de Goi as

38

Mostre que se A possui uma quantidade mpar de elementos, ent ao necessariamente, 1 A. Justique sua resposta. Solu c ao apresentada pela comiss ao da XIV OMEG. p p q Os elementos A para os quais = s ao em n umero par. Se A tem q q p p0 A deve satisfazer um n umero mpar de elementos, algum dos q0 q0 p0 = q0 p0 = p2 0 2 =1 q0 = p0 = 1, q0

por serem os elementos de A positivos. Problema 4) Escolha quaisquer quatro n umeros pares consecutivos (por exemplo: 6, 8, 10, 12). Multiplique os dois do meio (isto e, 8 10 = 80). Multiplique o primeiro e o u ltimo (isto e, 6 12 = 72). A seguir, subtraia os resultados obtidos (isto e, 80 72 = 8). Tente repetir isto com outros n umeros ` a sua escolha. Por que a resposta ser a sempre 8? Justique sua resposta. Solu c ao apresentada por Marcelo Abdala Daher: Chame os quatro n umeros pares consecutivos de x, x +2, x +4, x +6. Da temos: (x + 2)(x + 4) x(x + 6) = x2 + 6x + 8 (x2 6x) = 8. Problema 5) Um tri angulo e chamado de tri angulo de Heron se sua area e os comprimentos dos lados s ao ambos n umeros inteiros. Por outro lado, se os comprimentos dos lados s ao inteiros a, b, c satisfazendo a2 = b2 + c2 , o tri angulo e chamado de pitag orico, em homenagem a Pit agoras (570 a.C. - 490 a.C.). Por exemplo, o tri angulo de lados 3, 4 e 5 tem area 6 e, portanto, e de Heron. Ele tamb em e pitag orico, pois, 52 = 32 + 42 .

Olimp ada de Matem atica do Estado de Goi as

39

a) Mostre que o tri angulo T1 cujos lados medem 9, 12 e 15 e o tri angulo T2 cujos lados medem 5, 12 e 13 s ao pitag oricos e tamb em de Heron. b) Forme um novo tri angulo colando os tri angulos T1 e T2 do item a). Este tri angulo e pitag orico? e de Heron? Justique sua resposta. Solu c ao apresentada por Igor Muniz Soares e J ulio Ferreira Soares Filho: a) Para o tri angulo T1 : 92 + 122 = 81 + 144 = 152 e a area de T1 e 54. Logo T1 e pitag orico e de Heron. Para o tri angulo T2 : 52 + 122 = 25 + 144 = 169 = 132 . A area de T2 e 30. Logo e pitag orico e de Heron. 2 2 2 b) 15 = 225 = (5+9) +13 = 196+169 = 365 logo n ao e pitag orico. Sua area e 84 logo e de Heron. Problema 6) Pedro e Jo ao criaram um jogo de dardo cujo alvo circular tem tra cos c rculos conc entricos de raios diferentes. O objetivo e atirar dardos e acertar dentro dos c rculos. Sendo mais dif cil o acerto nos menores, e atribu da pontua c ao diferenciada para estes acertos: 5 pontos por acerto no c rculo menor; 3 pontos por acerto no anel intermedi ario (fora do c rculo menor e dentro do c rculo intermedi ario); ] 2 pontos por acerto no anel maior (fora do c rculo intermedi ario). Pedro e Jo ao estipularam ainda a seguinte regra: no primeiro acerto o jogador recebe a pontua c ao correspondente ` a posi c ao que seu dardo atingir o alvo e a partir do segundo acerto, sua pontua c ao ser a dobrada cada vez que ele acertar o anel maior; triplicada cada vez que ele acertar o anel intermedi ario e quintuplicada cada vez que ele acertar o c rculo menor. Ao m de uma dessas disputas, Pedro obteve 32400 e Jo ao 24000. Quem acertou mais vezes o c rculo menor? Justique sua resposta. Solu c ao apresentada por Andr e Santiago Beires: Para descobrir quem acertou mais vezes o c rculo menor basta fatorar a pontua c ao de cada um. Pois quando voc e acerta o c rculo menor voc e quintuplica seus pontos e quantas vezes der para dividir por 5 corresponde a quantas vezes ele acertou o c rculo menor. Logo, como

Olimp ada de Matem atica do Estado de Goi as

40

32400 = 24 34 52 e 2400 = 26 31 53 , temos que Jo ao foi quem acertou mais vezes o c rculo menor.

N vel 2
Problema 1) H a um rel ogio cujo mostrador teve seus n umeros embaralhados. Ache todos os n umeros a partir das dez arma c oes seguintes. 1. O n umero na posi c ao F est a correto. 2 A diferen ca entre os n umeros nas posi c oes G e F e 6. 3. A soma dos n umeros A e G e treze. 4. O n umero na posi c ao L e o dobro do n umero A, um ter co do n umero na posi c ao D e metade do n umero na posi c ao E. 5. Os n umeros D e J tamb em somam treze. 6. O n umero na posi c ao D e o qu adruplo de um dos n umeros adjacentes a ele. 7. O n umero na posi c ao D e o dobro do n umero na posi c ao H. 8. A soma dos seis primeiros n umeros (A a F ) e igual ` a soma dos seis u ltimos. 9. N ao h a nenhum par de n umeros consecutivos. 10. Nenhum n umero par est a entre dois n umeros mpares. Para a localiza c ao dos n umeros siga no mostrador abaixo, onde as letras de A a L foram utilizadas para facilitar o trabalho.

Solu c ao apresentada por Du lio Cardoso de S a: Se F est a na posi c ao certa, F = 5. Sendo a diferen ca entre F e G igual a 6, G e 11. Sendo A + G = 13, ent ao A = 2. Se L = 2A, E L = 4. Sendo L = D 3 , segue que D = 12 e se L = 2 segue que E = 8. Como D = 2H , segue que H = 6. Se D + J = 13, temos J = 1. Se D = 4E ou D = 4C , C ou E valer a 3, como E = 8 temos C = 3. Sendo A + B + C + D + E + F = G + H + I + J + K + L, podemos dizer que B + 30 = I + K + 22. Como faltam os n umeros 7, 9 e 10, B ser a igual a 9, pois 9 + 30 = 7 + 10 + 22 = 39. Falta descobrir entre I e K qual e 7 e qual e 10. Como I est a ao lado de H = 6, I n ao pode valer 7, j a

Olimp ada de Matem atica do Estado de Goi as

41

que n ao h a nenhum n umero par entre dois n umeros mpares. Portanto I = 10 e K = 7. Problema 2) Escolha quaisquer quatro n umeros pares consecutivos (por exemplo: 6, 8, 10, 12). Multiplique os dois do meio (isto e, 8 10 = 80). Multiplique o primeiro e o u ltimo (isto e, 6 12 = 72). A seguir, subtraia os resultados obtidos (isto e, 80 72 = 8). Tente repetir isto com outros n umeros ` a sua escolha. Por que a resposta ser a sempre 8? Justique sua resposta. Solu c ao apresentada por Alice Duarte Scarpa: Os quatro n umeros pares consecutivos ter ao a forma 2k, 2k + 2, 2k + 4, 2k + 6. Multiplicando os dois do meio, obtemos: (2k + 2)(2k + 4) = 4k2 + 12k + 8. Multiplicando o primeiro e o u ltimo segue que: 2k(2k + 6) = 4k2 + 12k. Subtraindo os dois termos temos: 4k2 + 12k + 8 (4k2 + 12k) = 8. Problema 3) Seja ABC um tri angulo ret angulo em B . Sobre o lado BC , construa o tri angulo equil atero BCD , conforme a gura. Seja Q o ponto m edio do lado AC . Mostre que os segmentos DQ e BA s ao paralelos.
C

Justique sua resposta Solu c ao apresentada por Pedro Victor Gomes de Oliveira: Se Q e o ponto m edio do lado AC , ele tamb em e o ponto m edio da altura do tri angulo ABC . Agora BC e altura do tri angulo ABC , e o tri angulo BCD e um tri angulo equil atero, logo a bissetriz de D passa pelo ponto m edio de BC , formando um angulo reto. Logo a reta QD passa pelo ponto m edio de BC formando um angulo reto. Portanto temos DQ formando angulo reto com BC e BC forma angulo reto com AB , logo DQ e paralelo a AB . Solu c ao apresentada pela comiss ao da XIV OMEG: Basta mostrar que DQ CB . Liguemos B a Q. Se mostrarmos BQ = CQ os tri angulos DCQ e DBQ ser ao congruentes e CM = M B

Olimp ada de Matem atica do Estado de Goi as

42

(com M = DQ CB ); da , portanto, DQ CB . Ligando Q a N , o ponto m edio de AB , v e-se, da , que os tri angulos QN A e CBA s ao semelhantes, com AN AQ QN 1 = = = , logo, BQ = CQ. AB AC CB 2 Problema 4) Pedro e Jo ao criaram um jogo de dardo cujo alvo circular tem tra cos c rculos conc entricos de raios diferentes. O objetivo e atirar dardos e acertar dentro dos c rculos. Sendo mais dif cil o acerto nos menores, e atribu da pontua c ao diferenciada para estes acertos: 5 pontos por acerto no c rculo menor; 3 pontos por acerto no anel intermedi ario (fora do c rculo menor e dentro do c rculo intermedi ario); 2 pontos por acerto no anel maior (fora do c rculo intermedi ario). Pedro e Jo ao estipularam ainda a seguinte regra: no primeiro acerto o jogador recebe a pontua c ao correspondente ` a posi c ao que seu dardo atingir o alvo e a partir do segundo acerto, sua pontua c ao ser a dobrada cada vez que ele acertar o anel maior; triplicada cada vez que ele acertar o anel intermedi ario e quintuplicada cada vez que ele acertar o c rculo menor. Ao m de uma dessas disputas, Pedro obteve 32400 e Jo ao 24000. Quem acertou mais vezes o c rculo menor? Justique sua resposta Solu c ao apresentada por Alice Duarte Scarpa: Vamos decompor as pontua c oes de Jo ao e Pedro em fatores de 2, 3, 5. Acertou mais vezes o c rculo menor aquele cuja pontua ca o possui o maior expoente da base 5. Como Pedro obteve 32400 = 24 34 52 e Jo ao 24000 = 26 31 53 , quem acertou mais vezes o c rculo menor foi Jo ao. Problema 5) Considere a express ao 3n + 7n , n N. Note que para 1 1 n = 1 tem-se 3 + 7 = 10. Determine os todos os valores de n > 1 para os quais 3n + 7n e divis vel por dez. Justique sua resposta. Solu c ao apresentada por Pedro Victor Aniz Gomes de Oliveira: Para que um n umero seja divis vel por dez ele tem que terminar em zero. Para que a soma de dois n umeros termine em zero a soma dos algarismos da unidade tem que ser igual a dez. Observemos os algarismos das unidades das pot encias de 3 e 7:

Olimp ada de Matem atica do Estado de Goi as

43

31 = 3, 32 = 9, 33 = 27, 34 = 81, 35 = 243, 1 2 3 4 7 = 7, 7 = 49, 7 = 343, 7 = 2401, 75 = 16807. Sempre que n e mpar o algarismo e 3 ou 7, de maneira alternada. Logo sempre que n for mpar temos o resultado desejado. Problema 6) Atribui-se a Heron a seguinte f ormula para o c alculo da area, A, de um tri angulo cujos lados medem a, b e c A= s(s a)(s b)(s c), com s= a+b+c . 2

Um tri angulo e chamado de tri angulo de Heron se sua area e os comprimentos dos lados s ao ambos n umeros inteiros. Por exemplo, o tri angulo de lados 3, 4 e 5 tem area 6 e, portanto, e de Heron. a) Verique que o tri angulo de lados 13, 14 e 15 e de Heron. b) Se os n umeros inteiros consecutivos b 1, b e b + 1 s ao lados de um tri angulo de Heron, mostre que b e um n umero par. Justique sua resposta. Solu c ao apresentada pela comiss ao da XIV OMEG: 1 ormula de Heron, a) Tem-se s = (13 + 14 + 15) = 21 e, pela f 2 A = 21 (21 13) (21 14) (21 15) = 24 32 72 = 84. b) Sejam b 1, b, b + 1 lados de um tri angulo de Heron. Ent ao s= 3b , 2 A= 3b 2 b 2 b+2 2 b2 2 = b 4 3(b2 4).

Note que se b for mpar, A n ao ser a um n umero inteiro pois, mesmo que 2 a um n umero mpar. b 3(b 4) seja inteiro, ser

N vel 3
Problema 1) Determine todos os valores de n N para os quais 3 | (1 + 2 + 22 + + 2n ). Justique sua resposta.

Olimp ada de Matem atica do Estado de Goi as

44

Solu c ao apresentada por Pedro Henrique Guedes de Oliveira: Temos que 3|(2x + 2x+1 ) pois 2x + 2x+1 = 2x + 2.2x = 3.2x . Assim sendo, para que 3 seja divisor de (20 + 21 + ... + 2n ) e necess ario que haja um n umero par de termos nessa soma. Logo temos que 3|(1+2+ ... +2n ) para n = 2k + 1, para k N.

Problema 2) Considere a express ao 3n + 7n , n N. Note que para 1 1 n = 1 tem-se 3 + 7 = 10. Determine todos os valores de n > 1 para os quais 3n + 7n e divis vel por dez. Justique sua resposta. Solu c ao apresentada por Luciana M. R. Salgado:

Para que um n umero seja divis vel por 10 seu algarismo das unidades deve ser zero. Observando que 31 = 3, 71 = 7, 32 = 9, 72 = 49, 33 = 27, 73 = 343, 74 34 = 81, = 2401, 35 = 243, 75 = 16807.

Vemos que as pot encias de 3 apresentam como algarismos das unidades a seq u encia (3, 9, 7, 1), enquanto as pot encias de 7 apresentam (7, 9, 3, 1). Assim temos como poss veis valores para n, uma Progress ao Aritm etica, cujo primeiro termo e 3 e de raz ao 2. Da n = 3 + (x 1)2 ou ainda n = 2x 1. Portanto os poss veis valores de n s ao aqueles que respeitam a lei de forma c ao n = 2x 1, x N e n > 1, ou seja, n mpar. Problema 3) Seja ABC um tri angulo ret angulo em B . Sobre o lado BC , construa o tri angulo equil atero BCD , conforme a gura. Seja Q o ponto m edio do lado AC . Mostre que os segmentos DQ e BA s ao paralelos.
C

Justique sua resposta. Solu c ao apresentada por Guilherme Victor Humberto Soares Carneiro: O tri angulo ABC e ret angulo, isso implica que as proje c oes do ponto m edio da hipotenusa nos catetos ser ao seus respectivos pontos m edios. O tri angulo CBD e equil atero, o que implica que a proje c ao do v ertice D na base CB ser a seu ponto m edio. Logo, as proje c oes de D e Q no segmento CB s ao o mesmo ponto que chamaremos M1 . Ou seja, DQ

Olimp ada de Matem atica do Estado de Goi as

45

passar a pelo ponto M1 formando um angulo reto com CB , j a que se trata de proje c oes. Sendo os dois segmentos AB e DQ perpendiculares a CB e pertencentes ao mesmo plano eles s ao paralelos. Solu c ao apresentada por Leonardo Augusto de Oliveira: Se ABC e um tri angulo circunscrito em uma circunfer encia tem-se Q como circuncentro, j a que e o ponto m edio da hipotenusa do tri angulo ABC . Portanto CQ = QA = BQ = raio. Sendo BCD um tri angulo equil atero, BD = CD . Como BQ = CQ, segue que DQ e a mediatriz do segmento BC . Se DQ e a mediatriz de BC , o angulo BOQ e reto, assim como o angulo OBA tamb em e reto. Portanto, sendo ambos os segmentos DQ e AB perpendiculares ao mesmo segmento BC , conclu mos que DQ e paralelo a AB . Problema 4) Atribui-se a Heron a seguinte f ormula para o c alculo da area, A, de um tri angulo cujos lados medem a, b e c A= s(s a)(s b)(s c), com s= a+b+c . 2

Um tri angulo e chamado de tri angulo de Heron se sua area e os comprimentos dos lados s ao ambos n umeros inteiros. Por exemplo, o tri angulo de lados 3, 4 e 5 e de Heron. a) Se os n umeros inteiros consecutivos b 1, b e b + 1 s ao lados de um tri angulo de Heron, mostre que b e um n umero par. b) Sejam b 1, b e b + 1 lados de um tri angulo de Heron, com b = 2x, x N. Mostre que existe y N tal que x2 3y 2 = 1. c) Generalize as conclus oes dos itens a) e b), supondo agora que os lados de um tri angulo de Heron formem uma PA de raz ao d = 0. Justique sua resposta. Solu c ao apresentada por Pedro Henrique Guedes de Oliveira:

Olimp ada de Matem atica do Estado de Goi as

46
a+b+c 2

a) Temos A = A= =

s(s a)(s b)(s c), como s =

segue que:

(a + b + c) (a + b c) (a b + c) (a + b + c) 2 2 2 2 3b.b.(b 2).(b + 2) 3b (b 2) b (b + 2) = . 2 2 2 2 4

Caso b fosse mpar, 3b, (b 2) e (b + 2) tamb em seriam, de modo que 3b.b.(b 2).(b + 2) seria um n umero mpar. Como a raiz de um n umero mpar e mpar ter amos que A seria a divis ao de um n umero mpar por 4, n ao podendo assim ser um n umero inteiro. Assim sendo, para que o tri angulo possa ser classicado como de Heron, b deve ser par. b) Como mostrado no item a: A= 3b2 (b2 4) = 4 = x 3(x2 1). 3(2x)2 ((2x)2 4) = 4 16.3x2 .(x2 1) 4

Para que A seja um n umero inteiro inteiro 3(x2 1) = k2 = x2


2

k2 = 1. 3

Tomando k = 3y temos k3 = 3y 2 e assim x2 3y 2 = 1. c) Sendo a = b d, c = b + d segue que A= 3b.b.(b 2d).(b + 2d) . 4

Problema 5) Sejam a, b R, a, b > 0 e a, b = 1. Tem-se logb a = r a = br . A mudan ca da base b para a base a e feita por interm edio da f ormula 1 . loga b = logb a

Temos que 2r e par. Se b e mpar temos que 3b, (b 2d) e (b + 2d) tamb em ser ao mpares. Desse modo, o produto destes n umeros e um n umero mpar assim como sua raiz, fazendo com que A n ao seja um n umero inteiro. Portanto, para que o tri angulo seja de Heron b deve ser par. Al em disso se b = 2x temos A = x 3(x2 d2 ). Para que A seja 2 n umero inteiro devemos ter 3(x2 d2 ) = k2 . Da x2 d2 = k3 . Tomando k = 3y segue que x2 3y 2 = d2 .

Olimp ada de Matem atica do Estado de Goi as

47 para todos b, c > 0.

a) Mostre que loga (bc) = loga b + loga c, b) Calcule a soma

1 1 1 1 + + + + . log2 2005 log3 2005 log4 2005 log2005 2005 Justique sua resposta. Solu c ao apresentada por Pedro Henrique Guedes de Oliveira: a) Como loga bc = x ax = bc, loga b = m b = am e loga c = n c = an .

Da segue que ax = am .an = am+n . Logo loga bc = m + n = loga b + loga c. b) 1 1 1 + + ... + = log2 2005 log3 2005 log2005 2005 = log2005 2 + log2005 3 + ... + log2005 2005, = log2005 (2.3.4. . . . .2005), = log2005 (2005!). Problema 6) Dado um ret angulo ABCD , e poss vel encontrar um ponto P em BC e um ponto Q em CD de modo que o tri angulo AP Q seja equil atero?
D
xxxxxxxxxxxxxxxxxxxxxx xxxxxxxxxxxxxxxxxxxxxx xxxxxxxxxxxxxxxxxxxxxx xxxxxxxxxxxxxxxxxxxxxx xxxxxxxxxxxxxxxxxxxxxx xxxxxxxxxxxxxxxxxxxxxx xxxxxxxxxxxxxxxxxxxxxx xxxxxxxxxxxxxxxxxxxxxx xxxxxxxxxxxxxxxxxxxxxx xxxxxxxxxxxxxxxxxxxxxx xxxxxxxxxxxxxxxxxxxxxx xxxxxxxxxxxxxxxxxxxxxx xxxxxxxxxxxxxxxxxxxxxx xxxxxxxxxxxxxxxxxxxxxx xxxxxxxxxxxxxxxxxxxxxx xxxxxxxxxxxxxxxxxxxxxx xxxxxxxxxxxxxxxxxxxxxx xxxxxxxxxxxxxxxxxxxxxx

P B

xxxxxxxxxxxxxxxxxxxxxx xxxxxxxxxxxxxxxxxxxxxx xxxxxxxxxxxxxxxxxxxxxx xxxxxxxxxxxxxxxxxxxxxx xxxxxxxxxxxxxxxxxxxxxx xxxxxxxxxxxxxxxxxxxxxx xxxxxxxxxxxxxxxxxxxxxx xxxxxxxxxxxxxxxxxxxxxx xxxxxxxxxxxxxxxxxxxxxx xxxxxxxxxxxxxxxxxxxxxx xxxxxxxxxxxxxxxxxxxxxx xxxxxxxxxxxxxxxxxxxxxx xxxxxxxxxxxxxxxxxxxxxx xxxxxxxxxxxxxxxxxxxxxx xxxxxxxxxxxxxxxxxxxxxx xxxxxxxxxxxxxxxxxxxxxx xxxxxxxxxxxxxxxxxxxxxx xxxxxxxxxxxxxxxxxxxxxx xxxxxxxxxxxxxxxxxxxxxx

P B

Justique sua resposta. Solu c ao apresentada pela comiss ao da XIV OMEG: Fa camos AB = CD = a, AD = BC = b, DQ = x < a e BP = y < b. Ent ao P C = b y , QC = a x. Os tri angulos ADQ, QCP, P BA s ao

Olimp ada de Matem atica do Estado de Goi as

48

tri angulos ret angulos cujas hipotenusas s ao AQ, P Q, AP , respectivamente. Queremos que AQ = P Q = AP, logo a2 +y 2 = x2 +b2 = (ax)2 +(by )2 = 2(ax+by ) = a2 +y 2 = x2 +b2 .

De x = (a2 + y 2 2by )/2a e x2 + b2 = a2 + y 2 temos y 2 4by + 4b2 3a2 = 0. Da , de 0 < y < b, conclui-se y = 2b a 3, e nalmente x = 2a b 3. Como devem ser a, b R para que existam os pontos procurados? Luciana Maria de Avila Rodrigues Endere co: Universidade Federal de Goi as Instituto de Matem atica e Estat stica Caixa Postal 131 74001-970 - Goi ania - GO - Brasil luavila@mat.ufg.br

Revista da Olimp ada - IME - UFG, no - 7, Setembro 2008,

49-61

Solu c oes Comentadas das Provas da XV OMEG - 2006


Edm eia Fernandes da Silva3 Resumo. Nesta se c ao apresentamos as solu c oes comentadas dos participantes da XV OMEG.

N vel 1
Problema 1) L ucio e Lucas s ao irm aos g emeos e, no u ltimo natal, ganharam rel ogios id enticos de uma tia. Lucas, muito met odico, sempre deixa seu rel ogio adiantado dez minutos, o que n ao adianta muito, porque ele leva isso em conta e chega aos seus compromissos pontualmente na hora marcada (e n ao 10 minutos antes). De qualquer forma ele acha divertido confundir as pessoas que pegam uma caronaolhando as horas em seu rel ogio. L ucio, embora muito pontual, j a e mais desligado, tanto que seu rel ogio est a cinco minutos atrasado e ele ainda n ao notou. De manh a, na correria para se arrumarem, os dois irm aos pegaram os rel ogios trocados sem perceber. Eles tinham combinado de encontrarem-se com o pai ao meio-dia para almo car. Supondo que a troca dos rel ogios n ao seja percebida, qual dos dois irm aos chegar a primeiro? Quanto tempo depois chegar a o outro irm ao? Justique sua resposta. Solu c ao apresentada por Mateus Mota Moraes, Matheus Felter Rocha, Pedro Augusto Machado Jorge: L ucio chegar a 11:50 ao almo co pensando ser 12:00. O rel ogio no bra co de Lucas marcar a 12:10 e ele acreditar a estar em seu hor ario, mas como o rel ogio de L ucio est a 5 minutos atrasado, na verdade s ao 12:15 hs. Portanto L ucio chegar a 25 minutos antes de Lucas.
Agradecemos o trabalho de digita c ao parcial do bolsista: Camila da Silva Rodrigues.
3

Olimp ada de Matem atica do Estado de Goi as

50

Problema 2) Um dado com forma de um cubo tem suas faces numeradas de 1 a 6. A gura abaixo mostra o mesmo dado em duas posi c oes diferentes. Qual e a face oposta ` a face 1?

Justique sua resposta. Solu c ao apresentada por Lucas Cardoso de Oliveira, Nath alia Gomes Mialichi e Ricardo Skrebsky Rubenich: Observe que o n umero oposto ` a face 1 e adjacente a face 5. Pela gura vemos que as faces adjacentes a face 5 s ao as faces 1,2,3 e 4, logo a face 6 e oposta ` a face 5. No dado 2 vemos que a face 3 e a face 5 s ao adjacentes a face 1. Logo a face oposta a face 1 n ao pode ser a 3 nem a 5. Restam ent ao como possibilidades as face 2 e 4. A face 4 n ao pode ser oposta ` a face 1 pois nesta situa c ao colocando os dois dados na mesma posi c ao, por exemplo com a face 1 para frente, no primeiro dado a face superior e 2 e no segundo dado a face superior e 3, o que n ao pode acontecer. Logo a u nica possibilidade e que a face oposta a ` face 1 seja a face 2. Problema 3) A colm eia da gura abaixo foi preenchida de acordo com uma certa regra e depois algumas casas foram apagadas. Descubra a regra e preencha o restante da colm eia.

A+B =C Justique sua resposta.

Olimp ada de Matem atica do Estado de Goi as

51

Solu c ao apresentada por Caio S. Carvalho, Matheus Felter Rocha: A regra e somar duas casas adjacentes para obter a casa de cima, como mostra a gura:

A + B = C.
3 na primeira linha, de baixo para Assim , por exemplo, para obter 4 cima, fazemos 9 3 1 2 = = . 12 3 12 4

Problema 4) Para a utiliza ca o em um certo jogo, s ao fabricadas cartas contendo, em um dos lados, um n umero e no outro lado uma cor, com a condi c ao de que, se uma carta tem um n umero par, seu verso deve ter a cor verde. Suponha que o fabricante suspeite de uma falha na fabrica c ao e pe ca para voc e examinar um conjunto de 4 cartas que est ao sobre uma mesa, para se certicar de que foram numeradas de acordo com a regra. As faces que voc e v e ao olhar para as cartas na mesa s ao,

Quais cartas voc e tem que virar (e basta examinar apenas elas) para saber se a regra foi obedecida? Justique sua resposta. Solu c ao apresentada por Giovanna Silva Bianchi, Lucas Cardoso de Oliveira e Nat alia Rodrigues Parrode:

Olimp ada de Matem atica do Estado de Goi as

52

Basta virar as cartas com o n umero 26 e com cor azul, porque o n umero 26 e par e deve ter o verso verde e a cor azul tem que ter o verso mpar. N ao e preciso virar as outras cartas, pois nem toda carta com a cor verde precisa ter verso par e sim toda carta com n umero par precisa ter verso verde. Problema 5) Um jogo consiste no seguinte: seu oponente pensa um n umero qualquer, de 1 a 8, e o seu objetivo e descobrir o n umero pensado, fazendo apenas perguntas cuja resposta pode ser sim ou n ao. a) Qual o menor n umero de perguntas que lhe garante acertar todas as vezes o n umero pensado? b) Se o n umero pensado fosse de 1 a 16, qual seria a resposta ` a pergunta do item a)? Justique sua resposta. Solu c ao apresentada por Leonardo Melo de Carvalho: Basta perguntar se o n umero e par ou mpar, reduzindo assim o n umero de possibilidades para a metade (4 ou 8). Depois perguntar se e maior ou menor que a metade do n umero dado, isto e 4 ou 8. Repita o processo at e que sobre apenas dois n umeros, depois e s o escolher um dos dois e perguntar se e este o n umero. Assim, no primeiro caso o menor n umero de perguntas e 3 e no segundo caso e 4. Problema 6) George e Juliana querem, cada qual, comprar um bombom. Para tanto, George precisa de mais 2 centavos e Juliana de mais 50 centavos. Se juntarem o dinheiro que possuem, ainda n ao ter ao o suciente para comprar um bombom. Qual o pre co deste bombom? Justique sua resposta. Solu c ao apresentada por Elias Achkar Filho: Para George s o faltam 2 centavos e se o dinheiro de George mais o de Juliana n ao e suciente para comprar o bombom e porque ela tem menos dinheiro que George. Temos ent ao as seguintes situa c oes: Se Juliana tem no m nimo 2 centavos, o bombom custa pelo menos 52 centavos e o George tem pelo menos 50 centavos e a soma do dinheiro dos dois e suciente para comprar o bombom, o que n ao e verdade. Se Juliana n ao tem dinheiro o bombom custa 50 centavos. Se Juliana tem 1 centavo, o bombom custa 51 centavos e George tem 49 centavos.

Olimp ada de Matem atica do Estado de Goi as

53

N vel 2
Problema 1) Jo ao fez um suco de caju misturando uma parte de polpa de caju com duas partes de agua. Como o suco cou forte e o recipiente j a estava cheio, Jo ao retirou um quarto do suco e completou o recipiente com agua. Ao nal, que fra ca o representava a quantidade de polpa de caju no suco? Justique sua resposta. Solu c ao apresentada por Cleuter Ant onio Pigorelli Carneiro e Ana Carina Peres Ferreira dos Santos: Representamos por c: caju, a: agua e r : total do suco. Temos Como 1 1 e, r temos Jo ao retira de suco, isto 4 4 3 c 2a 1 + r = r, 3 3 4 4 Assim, ou ou ainda mas r= c 2a + . 3 3

c 2a 1 c 2a 3 + ( + ) = r, 3 3 4 3 3 4 c 2a 2a 9 c ( ) ( + ) = r, 3 12 3 12 12

c 2a 3 + = r. 4 4 4 1 Completando o restante com agua, isto e, com de a gua, temos 4 3 1 c 2a 1a + + = r + r, 4 4 4 4 4 isto e, c 3a + = r. 4 4

3c 6a 9 + = r 12 12 12

1 3 de agua e de suco. 4 4 Problema 2) Um jogo consiste no seguinte: seu oponente pensa um n umero qualquer, de 1 a 8, e o seu objetivo e descobrir o n umero pensado, fazendo apenas perguntas cuja resposta pode ser sim ou n ao. a) Qual o menor n umero de perguntas que lhe garante acertar todas as vezes o n umero pensado? b) Se o n umero pensado fosse de 1 a 32, qual seria a resposta ` a pergunta do item a)? O suco tem

Olimp ada de Matem atica do Estado de Goi as

54

Justique sua resposta. Solu c ao apresentada por Vaniele Guimar aes Carvalho e Arthur Magalh aes de Oliveira: Vemos que se a quantidade de n umeros e 2n , o menor n umero de perguntas e exatamente n. Por exemplo, perguntando se o n umero escolhido e par ou mpar, reduzimos nossa escolha a 2n1 n umeros. Depois n 1 dividimos estes 2 n umeros em dois conjuntos e perguntamos em qual deles o n umero se encontra. Repetimos este processo at e que sobre apenas um conjunto com 2 n umeros e fazemos a pergunta nal. Assim, na parte a) a resposta e 3 e na parte b) a resposta e 5. Problema 3) George e Juliana querem, cada qual, comprar um bombom. Para tanto, George precisa de mais 2 centavos e Juliana de mais 50 centavos. Se juntarem o dinheiro que possuem, ainda n ao ter ao o suciente para comprar um bombom. Qual o pre co deste bombom? Justique sua resposta. Solu c ao apresentada por Marcela Rodrigues de Magalh aes, Yu Tzu Wu, Ana Carina Peres Ferreira dos Santos. Seja b o pre co do bombom, x o valor que George possui e y o valor que Juliana possui. Temos: x + 0, 02 = b, (1) y + 0, 5 = b, (2) x + y < b. (3) Substituindo (1) em (3) temos x + y < x +0, 02 o que nos d a y < 0, 02, isto e Juliana tem menos que 2 centavos. Assim se Juliana tem 1 centavo o bombom custa 51 centavos e se Juliana n ao tem dinheiro o bombom custa 50 centavos. Problema 4) Chame de ret angulo AP um ret angulo cuja area seja numericamente igual ao per metro. Por exemplo, o ret angulo de lados 5 e um ret angulo AP pois sua area e 25 = 10 5 medindo 10 e 2 2 e seu
5 2

per metro 25 = 2 10 +
5 2

10

a) Se x e y s ao lados de um ret angulo AP, como voc e pode obter y se voc e sabe o valor de x?

Olimp ada de Matem atica do Estado de Goi as

55

poss b) E vel obter um ret angulo AP com um dos lados medindo 7? E um ret angulo AP com um dos lados medindo 1? c) Encontre todos os ret angulos AP em que todos os lados s ao inteiros. Justique sua resposta. Solu c ao apresentada por Comiss ao da XV OMEG: a) Indicando por A e P a area e per metro, respectivamente, temos A = xy e P = 2(x + y ). Ent ao, impor A = P equivale a xy = 2(x + y ) ou y (x 2) = 2x, logo y= 2x x2 com x = 2.

b) Para x = 7, temos y = 14 a um ret angulo AP com essas 5 , logo h dimens oes. O mesmo n ao ocorre para x = 1, pois y = 2. c) Devemos ter x e y inteiros. Do item a) y= 2x 4 =2+ . x2 x2

Assim, para y ser inteiro, x 2 4. Logo, x = 3, 4 ou 6, correspondendo a y = 6, 4 ou 3, respectivamente. Dessa forma, os ret angulos AP que possuem lados inteiros t em lados 3 e 6 ou 4 e 4 (neste caso um quadrado). Problema 5) Para a utiliza ca o em um certo jogo, s ao fabricadas cartas contendo, em um dos lados, um n umero e no outro lado uma cor, com a condi c ao de que, se uma carta tem um n umero par, seu verso deve ter a cor verde. Suponha que o fabricante suspeite de uma falha na fabrica c ao e pe ca para voc e examinar um conjunto de 4 cartas que est ao sobre uma mesa, para se certicar de que foram numeradas de acordo com a regra. As faces que voc e v e ao olhar para as cartas na mesa s ao

Quais cartas voc e tem que virar (e basta examinar apenas elas) para saber se a regra foi obedecida? Justique sua resposta.

Olimp ada de Matem atica do Estado de Goi as

56

Solu c ao apresentada por Felipe Rodrigues Parrode, Arthur Magalh aes de Oliveira, Bruno Franco Bel em, Vaniele Guimar aes Carvalho. necess E ario virar a carta de n umero 26 e a carta escrito azul, pois a carta com o n umero 26 e par, logo deve ser verde no verso. A terceira carta n ao precisa ser virada pois em seu verso pode ter um n umero par ou mpar (o n umero par deve ter o verso verde, mas nem toda carta verde precisa ter o verso par). A quarta carta n ao precisa ser virada pois n ao importa a cor que tem no verso. Problema 6) Dentro um quadrado desenha-se o maior c rculo poss vel (que naturalmente vai tocar os lados do quadrado na metade de seus comprimentos). A seguir, dentro do c rculo, desenha-se o maior quadrado poss vel, com os lados paralelos aos do quadrado inicial, como se pode ver na gura abaixo.

Considere duas regi oes: o quadrado menor e a parte do quadrado maior que ca fora do quadrado menor. Qual das duas tem a maior area? Justique sua resposta. Solu c ao apresentada por Cleuter Ant onio Pigorelli Carneiro, J ulio Ferreira Soares Filho, Guilherme Rezende Barros. Seja r o raio do c rculo, l o lado do quadrado menor e L o lado do quadrado maior. A diagonal do quadrado menor e igual ao di ametro do c rculo. Assim, usando o teorema de Pit agoras temos 2l2 = (2r )2 . Portanto a area do quadrado menor e 2r 2 . Por outro lado, o di ametro do c rculo e igual ao lado do quadrado maior. Logo a area do quadrado maior e L2 = (2r )2 = 4r 2 . A area do quadrado maior que ca fora do quadrado menor e L2 2 2 2 l = 4r 2r . Portanto as duas areas s ao iguais.

Olimp ada de Matem atica do Estado de Goi as

57

N vel 3
Problema 1) George e Juliana querem, cada qual, comprar um bombom. Para tanto, George precisa de mais 2 centavos e Juliana de mais 50 centavos. Se juntarem o dinheiro que possuem, ainda n ao ter ao o suciente para comprar um bombom. Qual o pre co deste bombom? Justique sua resposta. Solu c ao apresentada por Amanda Guedes de Oliveira: Considerando G a quantidade de dinheiro (em centavos) que George possui, J a quantidade de centavos de Juliana e x o pre co de cada bombom temos x > G + J . Como G = x 2 e J = x 50 segue que x 2 + x 50 < x, isto e, x < 52. No entanto, como Juliana precisa de 50 centavos para comprar o bombom, conclu mos que o mesmo custa, no m nimo, 50 centavos. Logo 50 x e x < 52. Portanto o bombom custa 50 ou 51 centavos, dependendo se Juliana n ao tem nenhum centavo ou tem 1 centavo. Problema 2) Um jogo consiste no seguinte: seu oponente pensa um n umero qualquer, de 1 a 16, e o seu objetivo e descobrir o n umero pensado, fazendo apenas perguntas cuja resposta pode ser sim ou n ao. a) Qual o menor n umero de perguntas que lhe garante acertar todas as vezes o n umero pensado? b) Se o n umero pensado fosse de 1 a n, qual seria a resposta ` a pergunta do item a)? Justique sua resposta. Solu c ao apresentada por Jos e Carlos Carvalho J unior: Seja x o menor n umero de perguntas necess arias para descobrir um n umero de 1 a n. Este x e tal que 2x1 < n 2x . Assim se n = 16, devemos determinar x tal que 2x1 < 16 2x , o que implica em x = 4. (Para uma justicativa do fato acima, ver solu c ao da comiss ao). Solu c ao apresentada por Comiss ao da XV OMEG: Cada pergunta (com resposta sim ou n ao) divide o conjunto de n umeros que cont em o n umero pensado em dois subconjuntos. A resposta ` a pergunta revela em qual dos dois subconjuntos o n umero pensado est a. Em um dado momento do jogo se k e o n umero de possibilidades

Olimp ada de Matem atica do Estado de Goi as

58

restantes, seja P (k) o n umero m nimo de perguntas que lhe garante, ao nal, acertar o n umero pensado. Para um n umero pequeno de possibilidades, e f acil vericar, por exemplo, que P (2) = 1, P (3) = P (4) = 2, P (5) = P (6) = P (7) = P (8) = 3, e n ao e dif cil inferir que P (k) coincide com o expoente da menor pot encia de 2 que e maior ou igual a k. Uma maneira de provar esse resultado, e vericar o seguinte, onde todos os n umeros envolvidos s ao inteiros positivos: (1) r > s P (r ) P (s), (2) P (2r ) = r, (3) s > 2r P (s) > P (2r ).

A proposi c ao (1), embora possa ser demonstrada com mais rigor, e intuitivamente evidente: se voc e aumenta o n umero de possibilidades, vai precisar no m nimo do mesmo n umero de perguntas para chegar ao n umero pensado. A proposi c ao (2) tamb em e f acil de provar: se o n umero de possibilidades e uma pot encia de 2, isto e, 2r , a estrat egia que minimiza o n umero de possibilidades restantes a cada passo, como j a vimos, e fazer perguntas que, com certeza, reduzem ` a metade o n umero de possibilidades, a cada resposta. Assim, ap os respondida a primeira perr r 1 gunta restar ao (2 /2) = 2 possibilidades, a segunda pergunta deixa 2r2 possibilidades e assim sucessivamente, at e que a resposta ` a r - esima pergunta deixa 2rr = 1 possibilidades, isto e, leva ao n umero pensado. Para mostrar a proposi c ao (3), suponha que s = 2r + 1 e o n umero de possibilidades restantes (que e obviamente um n umero mpar). Fazendo uma pergunta que divida esse conjunto em dois subconjuntos, de mar 1 +1 neira a minimizar o tamanho do maior deles, o maior ter a s+1 2 =2 elementos. Na pior das hip oteses o n umero pensado estar a nesse maior subconjunto e uma segunda pergunta, analogamente, pode dividi-lo em dois conjuntos menores, o maior deles com 2r2 + 1 elementos. Procedendo de modo an alogo, no pior cen ario, ap os r perguntas respondidas, restar ao 2rr +1 = 2 possibilidades, e ser a necess aria mais uma pergunta para chegar ao n umero pensado. Portanto P (2r + 1) = r + 1 > P (2r ), pela proposi c ao (2). Por outro lado, se s > 2r + 1, pela proposi c ao (1), tem-se P (s) P (2r + 1) > P (2r ). Finalmente, das tr es proposi c oes acima, segue que se n for o n umero de possibilidades para o n umero pensado e r for o n umero inteiro positivo tal que 2r < n 2r+1 , ent ao r = P (2r ) < P (n) P (2r+1 ) = r + 1, ou seja P (n) = r + 1 e o menor n umero de perguntas que lhe garante chegar ao n umero pensado em qualquer cen ario, como quer amos mostrar.

Olimp ada de Matem atica do Estado de Goi as

59

Problema 3) Um monumento consiste de duas semi-esferas de raio r , apoiadas num mesmo plano horizontal, uma com a concavidade para cima (e conseq uentemente tangente ao plano em um ponto P ) e a outra com a concavidade para baixo (que conseq uentemente ap oia-se no plano numa circunfer encia com centro Q). cabo

Suponha que a dist ancia entre P e Q seja maior que 2r . Deseja-se estender um cabo horizontal retil neo conectando as duas superf cies e que seja o mais curto poss vel. Determine o comprimento desse cabo e sua altura em rela c ao ao plano de apoio. Justique sua resposta. Solu c ao apresentada por Let cia Goulart Netto e Thales Rodrigues Bosco: Considere a gura
t r rh rx x r P h Q y ry s

Seja o comprimento do cabo e P Q o comprimento do segmento P Q. Observando a gura acima vemos que = P Q(r x)(r y ). Note que, a medida que o cabo sobe, x diminui e y aumenta, e vice-versa. Assim, ser a m nimo quando x = y . Usando Teorema de Pit agoras obtemos 2 2 2 2 as seguintes equa c oes: r = (r h) + (r x) e r = h2 + (r x)2 , o r r que nos d a h = . Substituindo h = em h2 + (r x)2 = r 2 segue que 2 2 3 ). Como = P Q 2r + 2x, temos = P Q r 3. x = r (1 2 Problema 4) Encontre todos os ret angulos de lados inteiros cuja area e igual ao per metro. Justique sua resposta.

Olimp ada de Matem atica do Estado de Goi as

60

Solu c ao apresentada por Amanda Guedes de Oliveira e Daniel Soares Carneiro: Sejam x e y os lados do ret angulo, A sua area e P per metro. Temos A = xy e P = 2x + 2y . Ent ao xy = 2x + 2y . Assim xy 2x = 2y x(y 2) = 2y x =
2y y 2

2y . y2
2y y 2

y . Portanto, 2y y 2 2y , isto e, y (y 4) 0 o que implica 2x obtemos 2 < x 4. y 4. Por outro lado, escrevendo y = x2 Logo existem apenas 2 ret angulos inteiros cuja area e igual ao per metro. O quadrado de lado 4 e o ret angulo de lados 3 e 6. temos Problema 5) Na gura abaixo, ABCD e um quadrado e P e um ponto em seu interior.

Logo y 2 > 0, ou seja, y > 2. Consideremos x y , como x =

Mostrar que a desigualdade as medidas P A, P B e P C satisfazem ` P A + P C 2 P B. Justique sua resposta. Solu c ao apresentada por Comiss ao da XV OMEG: Comecemos fazendo a seguinte constru c ao: gire o tri angulo P BC em rela c ao ao v ertice B , de modo a identicar os pontos A e C . Isto dene um novo tri angulo de v ertices ABP , congruente ao tri angulo angulo com P BC , com AP = P C e BP = BP . Note a seguir que o tri v ertices P BP e um tri angulo ret angulo em B e tamb em is osceles, logo PP = angulo P AP 2 BP . Da desigualdade triangular, aplicada ao tri segue 2 BP = P P AP + AP = AP + P C . Problema 6) Mostre que para cada n umero primo mpar p, h a exatamente um inteiro positivo n tal que n(n + p) e um quadrado perfeito. Justique sua resposta.

Olimp ada de Matem atica do Estado de Goi as

61

Solu c ao apresentada por Comiss ao da XV OMEG: Demonstremos primeiro o seguinte resultado auxiliar: Se p, b, c s ao 2 2 2 inteiros positivos, com p primo e p + b = c , ent ao b e c s ao n umeros consecutivos. De fato, como p2 + b2 = c2 , temos p2 = c2 b2 = (c b)(c + b). Da e da primalidade de p resultam as seguintes possibilidades: ou i) p = c + b = c b ou ii) p2 = c b e 1 = c + b ou ainda iii) p2 = c + b e 1 = c b. Devemos descartar i) por implicar em b = 0; tamb em descartamos ii) pois b, c > 0 implica b + c 2. Portanto vale iii) implicando c = b + 1. A seguir, impondo n(n + p) = q 2 chega-se a n= p + p2 + 4q 2 . 2

Como n e um n umero inteiro segue que p2 + (2q )2 = s2 para algum inteiro s > 0, o qual, pelo resultado auxiliar, deve cumprir s = 2q + 1. Daqui obt em-se: n= p2 1 2
2

n+p=

p2 + 1 2

Edm eia Fernandes da Silva Endere co: Universidade Federal de Goi as Instituto de Matem atica e Estat stica Caixa Postal 131 74001-970 - Goi ania - GO - Brasil edmeia@mat.ufg.br

Revista da Olimp ada - IME - UFG, no - 7, Setembro 2008,

62-76

Solu c oes Comentadas das Provas da XVI OMEG - 2007


Edm eia Fernandes da Silva4 Resumo. Nesta se c ao apresentamos as solu c oes comentadas dos participantes da XVI OMEG.

N vel 1
Problema 1) Meu cachorro, o Costelinha, consome exatamente 3 latas de comida de cachorro a cada 4 dias. Vou viajar por duas semanas e deixar o Costelinha com minha tia. Quantas latas de comida, no m nimo, eu deveria deixar com ela para esse per odo? Justique sua resposta. Solu c ao apresentada por Isabela L. Costa, Cassio L azaro Barros Cabral, Paula Santana Marra: Se em 4 dias Costelinha consome 3 latas de comida, o seu consumo 3 = 0, 75 latas. Viajando por duas semanas, ou 14 dias, di ario e de 4 3 Costelinha consumir a 14 = 10, 5 latas de comida e portanto devo 4 deixar com a vizinha 11 latas. Problema 2) O ndice de massa corporal (IMC) e freq uentemente usado para determinar se uma pessoa adulta est a acima do peso. O IMC e calculado dividindo o peso, em kg, pelo quadrado da altura em metros, ou seja, peso IMC = . (altura)2 Pelo crit erio da Organiza c ao Mundial da Sa ude, se o IMC for maior que 25, a pessoa e considerada acima do peso.
Agradecemos o trabalho de digita c ao parcial das bolsistas: Lays Grazielle Cardoso Silva e Wanessa Moreira dos Santos.
4

Olimp ada de Matem atica do Estado de Goi as

63

Marcelo tem 2m de altura e pesava 120kg quando resolveu passar um tempo em um spa. Nos 4 meses em que cou no spa, ele conseguiu 1 a engordou 16kg. perder de seu peso original, mas depois que saiu j 3 (a) Qual e o atual IMC do Marcelo? (b) Supondo que o Marcelo perdeu a mesma quantidade de peso em cada m es que permaneceu no spa, em quanto tempo, depois de entrar, ele deixou de estar acima do peso? Justique sua resposta. Solu c ao apresentada por Isabela Oliveira Caldeira: 1 de seu peso, o que a) Marcelo pesava 120kg e no spa emagreceu 3 corresponde a 40kg, passando a pesar 80kg, mas voltou a engordar 16kg 96 cando com 96kg e seu IMC atual e 2 = 24. 2 b) Antes de entrar no spa o IMC de Marcelo era 30 e para n ao estar acima do peso o seu IMC deve ser menor ou igual a 25. Para saber qual x = 25 onde x representa o peso de o seu peso resolve-se a equa c ao 4 Marcelo, obtendo-se x=100kg. Como ele perdeu 40kg em 4 meses, ou 10kg por m es, ap os 2 meses ele perdeu 20kg chegando a 100kg. Problema 3) Na seq uencia de padr oes quadriculados da gura abaixo, cada padr ao e formado por varetas, todas de igual comprimento. O padr ao 2 utiliza seis varetas id enticas para formar 4 quadrados menores.

Padr ao 1

Padr ao 2

Padr ao 3

(a) Seguindo este modelo, quantas varetas seriam necess arias para formar um quadriculado com 64 quadradinhos? (b) E para formar um quadriculado com 900 quadradinhos?

Olimp ada de Matem atica do Estado de Goi as

64

Justique sua resposta. Solu c ao apresentada por Arthur Moraes do Lago, Yuri Luiz Dias Martins, Pedro Augusto Machado: Para formar um quadriculado de n n = n2 quadradinhos s ao necess arias 4 varetas para formar o quadrado externo, (n 1) varetas na vertical para dividir o quadrado em n partes e (n 1) varetas na horizontal para dividir o quadrado em mais n partes formando assim o quadriculado desejado. Desta forma o n umero necess ario de varetas e 4 + 2(n 1). Para 64 quadradinhos s ao necess arias 2(8 + 1) = 18 varetas e para 900 quadradinhos 2(30 + 1) = 62 varetas. Problema 4) Numa festa haviam dois bolos de chocolate de mesma espessura (altura), um quadrado e o outro circular, colocados na mesa sobre um forro xadrez. A gura abaixo mostra a mesa vista de cima e as linhas que formam o padr ao xadrez s ao igualmente espa cadas umas das outras. Cada bolo foi cortado em peda cos iguais, como mostram as linhas tracejadas. Marcelo, muito esperto e fan atico por bolo de chocolate, assim que viu a mesa j a sabia de qual dos dois bolos ia pegar um peda co. Explique qual dos dois bolos tem os maiores peda cos.

Justique sua resposta. Solu c ao apresentada por Let cia Duarte Rosique, Roberta Sud ario Pinheiro, Jo ao Victor Barbosa Paiva: O bolo quadrado ocupa totalmente 36 quadradinhos da toalha, enquanto que o bolo redondo n ao ocupa totalmente os 36 quadradinhos. Logo o peda co do bolo quadrado e maior. Problema 5) Imagine que diante de voc e h a tr es caixas de papel ao fechadas contendo frutas. Cada caixa tem uma etiqueta de identica c ao

Olimp ada de Matem atica do Estado de Goi as

65

e numa delas est a escrito MAC AS, em outra est a escrito LARAN E LARANJAS. Voc JASe na terceira MAC AS e sabe que as etiquetas est ao trocadas de forma que nenhuma das caixas est a rotulada corretamente e seu objetivo e redistribuir as etiquetas de maneira correta. Para isso voc e vai poder escolher uma das caixas e dela ser a sorteada apenas uma fruta, que ser a mostrada a voc e. Qual das caixas voc e escolhe e como faz para reorganizar as etiquetas? Justique sua resposta. Solu c ao apresentada por Let cia Milhomem Bueno, Isabela L. Costa, Thiago Yoshibatau Kuuval: E LARANJAS pois Escolho a caixa com a etiqueta de MAC AS como nenhuma caixa est a com a etiqueta certa nesta caixa tem apenas um tipo de fruta. Se sair uma ma c a, nesta caixa deve ser colocada a etiqueta MAC AS. A caixa com a etiqueta de LARANJAS e na verdade a caixa de ma c as e laranjas e por m a caixa com etiqueta MAC AS cont em apenas laranjas. Se sair laranja da caixa com eti E LARANJAS, esta caixa s queta MAC AS o tem laranjas, a caixa com etiqueta MAC AS tem laranjas e ma c as e a caixa com etiqueta LARANJAS s o tem ma c as. Problema 6) Entre os 100 alunos do 6o - ano de uma escola, 86 gostam de futebol e 68 gostam de v olei. (a) No m aximo quantos alunos n ao gostam de v olei nem de futebol? (b) No m nimo quantos alunos gostam tanto de v olei quanto de futebol? (c) Se entre os alunos houver exatamente 6 que n ao gostam de v olei nem de futebol, mas gostam de basquete e 4 que n ao gostam de nenhum destes tr es esportes, quantos alunos gostam tanto de v olei quanto de futebol? Justique sua resposta. Solu c ao apresentada por Isabela Oliveira Caldeira, Lucas C. Daher: a) Se todos os 68 alunos que gostam de v olei tamb em gostam de futebol, temos no m aximo 100 86 = 14 alunos que n ao gostam nem de v olei nem de futebol.

Olimp ada de Matem atica do Estado de Goi as

66

b) No m aximo (86 + 68) 100 = 54 alunos gostam tanto de v olei quanto de futebol. c)Se 10 alunos n ao gostam nem de v olei nem de futebol, 90 alunos gostam de v olei e/ou futebol. Como 86 alunos gostam de futebol, temos 4 alunos que gostam apenas de v olei e portanto dos 68 alunos que gostam de v olei, 64 tamb em gostam de futebol.

N vel 2
Problema 1) O gr aco em forma de pizza na gura abaixo representa a distribui c ao da mesada entre tr es crian cas. Cris recebeu um real a mais que Bia, que recebeu dois reais a mais que Artur. Quanto cada um recebeu?
Bia Cris Artur

Justique sua resposta. Solu c ao apresentada por Jo ao Paulo Yoshio da Silva: Se Artur recebeu x reais, ent ao Bia recebeu x + 2 reais e Cris recebeu x +3 reais. Observando o gr aco vemos que Artur recebeu 25% do total. Como x + 2 + x + 3 + x corresponde a 100% do total e 3x corresponde a 75%, 5 corresponde a 25%, isto e, x = 5. Problema 2) Na seq u encia de padr oes quadriculados da gura abaixo, cada padr ao e formado por varetas, todas de igual comprimento. O padr ao 2 utiliza seis varetas id enticas para formar 4 quadrados menores.

Padr ao 1

Padr ao 2

Padr ao 3

Olimp ada de Matem atica do Estado de Goi as

67

(a) Seguindo este modelo, quantas varetas seriam necess arias para formar um quadriculado com 64 quadradinhos? (b) E para formar um quadriculado com dez mil quadradinhos? (c) Qual e a rela c ao entre o n umero de quadradinhos e a quantidade de varetas? Justique sua resposta. Solu c ao apresentada por Yuri Rezende Souza: Para cada padr ao p que forma um quadriculado com q = p2 quadradinhos, precisamos de (p +1) varetas no sentido horizontal e (p +1) varetas no sentido vertical. Logo o n umero de varetas v e igual a 2(p + 1), isto e, v = 2(p + 1) = 2( q + 1). Assim, para 64 quadradinhos p = 64 = 8 e v = 2(8 + 1) = 18 varetas. Para 10000 quadradinhos p = 10000 = 100 e v = 202. Problema 3) Entre os 100 alunos do 8o ano de uma escola, 86 gostam de futebol e 68 gostam de v olei. (a) No m aximo quantos alunos n ao gostam de nenhum dos dois esportes? (b) Suponha que os 100 alunos estejam sentados em um audit orio e recebam as seguintes instru c oes, nesta ordem: 1) Os que gostam de futebol, quem de p e; 2) Os que gostam de v olei, tamb em quem de p e; 3) Os que n ao gostam de futebol, sentem-se; 4) Os que n ao gostam de v olei, sentem-se. Ao nal qual ser a maior, o n umero de alunos sentados ou o n umero de alunos em p e? (c) Se entre os alunos houver exatamente 6 que n ao gostam de v olei nem de futebol, mas gostam de basquete e 4 que n ao gostam de nenhum destes tr es esportes, quantos alunos gostam tanto de v olei quanto de futebol? Justique sua resposta. Solu c ao apresentada por Roberto S. Castro e Yuri Rezende Souza. :

Olimp ada de Matem atica do Estado de Goi as

68

a) Se todos os 68 alunos que gostam de v olei tamb em gostam de futebol, ent ao 100-86=14 e o m aximo de alunos que n ao gostam de nenhum dos dois esportes. b) Se os alunos seguirem as instru c oes dadas nessa ordem, car ao em p e apenas os alunos que gostam dos dois esportes, pois os que gostam apenas de v olei ou de futebol se sentar ao nas instru c oes 3 e 4, e os que n ao gostam de nenhum esporte nem se levantar ao. Com as instru c oes 1 e 2, no m aximo 86 alunos car ao de p e. Com as instru c oes 3 e 4 se sentar ao 100 68 = 32 alunos. Ent ao o n umero m aximo de alunos em 100 , o n umero p e ser a 100 (100 86) (100 68) = 54. Como 54 > 2 de alunos em p e e maior. c) Como 10 alunos n ao gostam nem de v olei nem de futebol, 90 alunos gostam de v olei ou de futebol. Assim o m aximo de alunos que gostam de v olei e de futebol e 86 + 68 90 = 64. Problema 4) Imagine que diante de voc e h a tr es caixas de papel ao fechadas contendo frutas. Cada caixa tem uma etiqueta de identica c ao e numa delas est a escrito MAC AS, em outra est a escrito LARAN E LARANJAS. Voc JASe na terceira MAC AS e sabe que as etiquetas est ao trocadas de forma que nenhuma das caixas est a rotulada corretamente e seu objetivo e redistribuir as etiquetas de maneira correta. Para isso voc e vai poder escolher uma das caixas e dela ser a sorteada apenas uma fruta, que ser a mostrada a voc e. Qual das caixas voc e escolhe e como faz para reorganizar as etiquetas? Justique sua resposta. Solu c ao apresentada por Marcelo Abdala Daher, Ricardo Skrebsky Rubenich e Isadora Carvalho M. Francescantanio.: E LARANJAS, pois A caixa escolhida e a com a etiqueta MAC AS ela tem apenas um tipo de fruta j a que est a etiquetada errada. Sejam x e y as frutas. Se da caixa escolhida retiro a fruta x, etiqueto a caixa com esta etiqueta. A caixa com a etiqueta da fruta y certamente esta errrada e portanto contem os dois tipos de frutas recebendo a etiqueta x e y e por m a caixa com a etiqueta x e y s o contem a fruta y , recebendo ent ao esta etiqueta.
3 tem uma propriedade interessante: sua Problema 5) Os n umeros 3 e 2 3 soma e seu produto s ao iguais, ou seja, 3 + 3 2 = 3 2 = 9/2.

Olimp ada de Matem atica do Estado de Goi as

69

(a) Dado um n umero a qualquer, para quais valores de a existe um n umero b que tanto somado quanto multiplicado por a d a o mesmo resultado? (b) Esta propriedade tamb em pode ser encontrada em somas e produtos com um n umero maior de parcelas. Por exemplo, e poss vel encontrar c tal que 3+ e d tal que 3+ 3 9 81 3 9 81 + + =3 . 2 7 d 2 7 d 3 9 3 9 + =3 2 c 2 c

Encontre dois n umeros naturais e e f tais que: 3+ e 3 9 81 e 3 9 81 + + + =3 2 7 67 f 2 7 67 f Justique sua resposta. Solu c ao apresentada por Nat alia Ara ujo Ferreira e Yuri Rezende Souza: a) Sejam a + b = a b = x. Assim b = x a e a(x a) = x, isto e, a2 ax + x = 0 ou ainda x(1 a) = a2 . Portanto se a = 1 temos x= b) a2 a1 e b= a . a1

3 3 81 e 3 3 9 81 e + + + = ou, equivalentemente, 1 2 67 f 1 2 7 67 f e 6561 e 6561 + = . Pelo item a) 938 f 938 f e = f


6561 938 6561 938

6561 . 5623

Logo e = 6561 = (81)2 e f = 5623 = 6561 938. Problema 6) De uma folha de papel quadrada, com 1m de lado, desejase recortar uma faixa retangular de 10cm de largura e que tenha o maior comprimento poss vel.

Olimp ada de Matem atica do Estado de Goi as

70

(a) Qual e esse comprimento m aximo? (b) Se a largura da faixa desejada fosse 45cm, qual seria o maior comprimento poss vel para a faixa? (Obs.: Se necess ario, considere que 2 vale aproximadamente 1, 4142.) Justique sua resposta. Solu c ao apresentada por Jo ao Paulo Yoshio da Silva: a) A diagonal do quadrado e maior do que seu lado. Assim, fazendo a faixa ao longo da diagonal temos 2 tri angulos ret angulos is osceles um em cada ponta da faixa com a hipotenusa igual a 10cm. Deste modo o x lado x deste tri angulo pode ser obtido da seguinte forma: sen 45 = , 10 onde x e um dos catetos e 10 e a hipotenusa. Portanto x = 5 2. Temos tamb em um tri angulo ret angulo is osceles cuja hipotenusa y 2. Logo tem comprimento da faixa e os catetos t e m comprimento 100-5 y 2 = 2(100 5 2)2 , ou y = 2(100 5 2). Portanto y = 131, 42cm. Se a faixa fosse cortada paralela ao lado do quadrado esta teria 1m de comprimento e n ao seria a maior poss vel. b) Neste caso, se a faixa for feita ao longo da diagonal obteremos dois tri angulos ret angulos is osceles, um em cada ponta, de hipotenusa 45 cm = 22,5cm. 45cm e altura 2 Como a diagonal do quadrado mede 100 2cm = 141,42cm, ao subtrairmos da diagonal 45cm que corresponde a soma das alturas do dois tri angulos da ponta, vemos que a faixa tem comprimento menor do que 1m, logo, neste caso a faixa ter a maior comprimento se for cortada paralelo a um dos lados, ou seja, 1m de comprimento.

N vel 3
Problema 1) Um copo no formato de um cilindro circular reto, com 15cm de altura e 10cm de di ametro, cont em suco pela metade de sua capacidade. No m aximo quantos cubos de gelo com 2cm de aresta podem ser colocados no copo sem que o suco derrame? Justique sua resposta. Solu c ao apresentada por Douglas Souza Soares, Leandro Sabocinski Castro e Rafael Elias Nascimento:

Olimp ada de Matem atica do Estado de Goi as

71

Considerando que os cubos de gelo n ao possam ser quebrados temos que calcular quantos cubos podem ser colocados lado a lado. Como a base do copo e circular de di ametro 10cm e cada aresta do cubo de gelo mede 2cm, podemos fazer camadas de 12 cubos de gelo. Cada camada com 12 cubos de gelo ocupa um volume de 12 8 = 96cm3 , onde 8cm3 e o volume de cada cubo de gelo. O volume total do copo e Vc = (52 ) 15 = 375cm3 ,
3 mas o copo tem suco at e sua metade, ou seja, o copo tem 375 2 cm = 588, 8cm3 de suco, restando ser preenchido os outros 588,8cm3 de volu588, 8 = me. O n umero de camadas de gelo para preencher tal volume e 96 3 6, 13. Mas 6 camadas preenchem 6 96 = 576cm do volume restando 588,8 - 576=12,8cm3 a serem preenchidos. Como um cubo tem 8cm3 , o m aximo de cubos de gelo que podemos colocar no copo sem que o suco transborde e 6 12 + 1 = 73 cubos de gelo.

Problema 2) Os n umeros 3 e 3 2 tem uma propriedade interessante: sua 3 9 soma e seu produto s ao iguais, ou seja, 3 + 3 2 = 3 2 = 2. (a) Dado um n umero a qualquer, ser a que sempre existe um n umero b que tanto somado quanto multiplicado por a d a o mesmo resultado? (b) Esta propriedade tamb em pode ser encontrada em somas e produtos com um n umero maior de parcelas. Por exemplo, e poss vel encontrar c tal que 3+ e d tal que 3+ 3 9 81 3 9 81 + + =3 . 2 7 d 2 7 d 3 9 3 9 + =3 2 c 2 c

Encontre dois n umeros naturais e e f tais que: 3+ 3 9 81 e 3 9 81 e + + + =3 . 2 7 67 f 2 7 67 f

Olimp ada de Matem atica do Estado de Goi as

72

Justique sua resposta. Solu c ao apresentada por Fl avio J unior R. Mendes: a a) Como a + b = ab temos a = b(a 1), isto e, b = , desde que a1 a = 1. b) Armamos que dados a, b, c, d n umeros quaisquer existe um n umero x tal que a + b + c + d + x = abcdx. De fato, a + b + c + d = abcdx x a + b + c + d = x(abcd 1), isto e, x= que a+b+c+d , desde que abcd = 1. abcd 1
9 7

3 ,c= Sejam a = 3, b = 2

ed=

81 67 .

Devemos encontrar x =

e f

tal

e 3 9 81 e 3 9 81 + + + =3 . 2 7 67 f 2 7 67 f Pelo que vimos acima 3+


9 81 3+ 3 6561 e 812 2 + 7 + 67 = = . = 3 9 81 2 f 81 2 7 67 5623 3 2 7 67 1

Portanto, e = 6561 e f = 5623. Problema 3) Entre os 100 alunos do ensino m edio de uma escola, 86 gostam de futebol e 68 gostam de v olei. (a) No m aximo quantos alunos n ao gostam de nenhum dos dois esportes? (b) Suponha que os 100 alunos estejam sentados em um audit orio e recebam as seguintes instru c oes, nesta ordem: 1) Os que gostam de futebol, quem de p e; 2) Os que gostam de v olei, tamb em quem de p e; 3) Os que n ao gostam de futebol, sentem-se; 4) Os que n ao gostam de v olei, sentem-se. Ao nal qual ser a maior, o n umero de alunos sentados ou o n umero de alunos em p e? (c) Se entre os alunos houver exatamente 6 que n ao gostam de v olei nem de futebol, mas gostam de basquete e 4 que n ao gostam de nenhum destes tr es esportes, quantos alunos gostam tanto de v olei quanto de futebol?

Olimp ada de Matem atica do Estado de Goi as

73

Justique sua resposta. Solu c ao apresentada por Eduardo Rodrigues Silva Filho, Andr e Lemes de Freitas, Matheus Alves Farah e C assio Sobocinski Castro: a) Para ter o m aximo de alunos que n ao gostam de nenhum dos esportes, todos os alunos que gostam de v olei devem gostar tamb em de futebol, e neste caso 100 86 = 14 e o n umero procurado. b) No nal car ao de p e apenas aqueles alunos que gostam de futebol e v olei. Seja x o total destes alunos, isto e, n umero de alunos que n ao gostam de nem de v olei e nem de futebol e seja y o n umero de alunos que n ao gostam de nenhum dos dois esportes. Como visto no item a) o valor de y varia entre 0 (zero) e 14. Temos que 86 x e o n umero de alunos que gostam de futebol e 68 x e o n umero de alunos que gostam de v olei. Assim, (86 x) + (68 x) + x + y = 100 154 x + y = 100. Portanto x = 50 + y . Como 0 y 14, temos 54 x 68, isto e, o n umero de alunos em p e e maior do que o n umero de alunos sentados. c) Do item b) temos a equa c ao (86 x) + (68 x) + x + y = 100. Neste caso y = 10 e x = 54 + y = 54 + 10 = 64. Problema 4) Imagine que diante de voc e h a tr es caixas de papel ao fechadas contendo frutas. Cada caixa tem uma etiqueta de identica c ao e numa delas est a escrito MAC AS, em outra est a escrito LARAN E LARANJAS. Voc JASe na terceira MAC AS e sabe que as etiquetas est ao trocadas de forma que nenhuma das caixas est a rotulada corretamente e seu objetivo e redistribuir as etiquetas de maneira correta. Para isso voc e vai poder escolher uma das caixas e dela ser a sorteada apenas uma fruta, que ser a mostrada a voc e. Qual das caixas voc e escolhe e como faz para reorganizar as etiquetas? Justique sua resposta. Solu c ao apresentada por Pedro Henrique P. Daco e M arcio Andr e de Godoy Uema: Como todas as etiquetas est ao trocadas, escolho a caixa com etiqueta E LARANJAS pois n MAC AS ao haver a possibilidade de existir duas frutas diferentes nessa caixa. Retirando uma fruta dessa caixa descubro qual a verdadeira etiqueta a ser colocada. Restam ent ao 2 caixas, uma

Olimp ada de Matem atica do Estado de Goi as

74

com etiqueta e outra sem. Como as etiquetas est ao erradas, tiro a etiqueta da caixa etiquetada e passo para a que est a sem etiqueta e coloco E LARANJAS que est a etiqueta MAC AS a em m aos, na caixa que cou sem etiqueta. Problema 5) De uma folha de papel quadrada, com 1m de lado, desejase recortar uma faixa retangular de 10cm de largura e que tenha o maior comprimento poss vel. (a) Qual e esse comprimento m aximo? (b) Se ao inv es da faixa voc e quisesse recortar da folha quadrada o maior semi-c rculo poss vel (metade de um c rculo), qual seria a area desse semi-c rculo? (Obs.: Se necess ario, considere que 2 vale aproximadamente 1, 4142.) Justique sua resposta. Solu c ao apresentada por Comiss ao da XVI OMEG: (a) O segmento de reta mais longo contido no quadrado e sua diagonal e, se a faixa e sucientemente estreita, e intuitivo pensar que ser a a mais longa poss vel se for recortada ao longo da diagonal como mostra a gura abaixo. Neste caso, seu comprimento ser a igual ao comprimento da diagonal do quadrado menos a soma das alturas dos dois tri angulos menores nas extremidades. Estes ser ao tri angulos ret angulos is osceles e considerando cada um deles como metade de um quadrado, e f acil deduzir que sua altura e metade da largura da faixa retangular, ou seja, 5cm. A diagonal do quadrado mede 2 1, 4142m ou 141,42cm. Subtraindo os 10cm que correspondem ` as alturas dos dois pequenos tri angulos, conclui-se que a faixa ca com 131,42cm de comprimento.

(b) As situa c oes poss veis est ao representadas nas guras 1 e 2.

Olimp ada de Matem atica do Estado de Goi as

75

r a/2

r r o2 Figura 2

A gura 2 representa o semi-c rculo de maior di ametro poss vel. De fato, da gura 1 obtemos a2 + b2 = (2r )2 e a + r = 1 (a largura do quadrado) 2

Da segunda equa c ao tiramos, r = 1 a ao, quanto menor o valor 2 , e ent de a maior o raio que conseguimos. Por outro lado, substituindo r na primeira equa c ao obtemos b2 = 4(1 a), mostrando que quanto menor o valor de a, maior ser a b. Mas h a ainda outra restri c ao, pois n ao queremos que o topo do semi-c rculo ultrapasse o lado de cima do quadrado, ou b + r 1 que implica b a. Portanto, seja, temos que exigir que 2 devemos ter a o menor poss vel e b o maior poss vel com b a, o que leva a b = a, que e a solu c ao apresentada na gura 2. No caso da gura 2, o raio satisfaz r 2 + r = 1 r = 2 2 0,586 . 2 2 = (3 2 Assim, a area do maior semi-c rculo poss vel e r 2) . 2 Problema 6) Numa certa turma foram distribu dos cart oes em branco, um para cada aluno. Os alunos foram, ent ao, orientados a escrever no cart ao um n umero real qualquer, de sua escolha. Depois os cart oes foram recolhidos em uma caixa e, calculando-se a soma dos valores escritos nos cart oes o resultado foi 20. A soma dos tr es menores foi 5 e a soma dos tr es maiores foi 7. Quantos alunos tem a turma? Justique sua resposta.

9
a

Figura 1

Olimp ada de Matem atica do Estado de Goi as

76

Solu c ao apresentada por Davi de Castro Silva: Se somarmos os 3 menores valores e os 3 maiores valores obtemos 12, logo faltam 8 para obter o resultado nal, e o n umero de alunos devem ser maior que 6. Seja n o n umero de alunos cuja soma dos cart oes e 8. Se n 3, ent ao tr es ou menos cart oes teriam soma 8, o que e imposs vel pois a soma dos 3 maiores valores e 7. Se n 6, ent ao a soma de tr es desses cart oes seria menor que 5, o que imposs vel. Logo n = 4 ou n = 5. Se n = 5, a soma de 3 desses cart oes e um resultado maior ou igual a 5 e menor ou igual a 7, e a soma dos outros dois cart oes esta entre 1 e 3, o que e imposs vel pois cada um desses n cart oes tem valor maior ou 7 umero de alunos e igual a 5 3 e menor ou igual a 3 . Portanto n = 4 e o n 10. Edm eia Fernandes da Silva Endere co: Universidade Federal de Goi as Instituto de Matem atica e Estat stica Caixa Postal 131 74001-970 - Goi ania - GO - Brasil edmeia@mat.ufg.br

Revista da Olimp ada - IME - UFG, no - 7, Setembro 2008,

77-83

O Teorema das Quatro Cores

Ronaldo Antonio dos Santos Resumo. Por mais que pare ca ing enuo colorir mapas, foi um problema ligado ` a colora c ao que contribuiu para o desenvolvimento de uma nova area em matem atica, a Teoria dos Grafos, e ainda lan cou a discuss ao sobre a validade de demonstra c oes que usam o computador de forma essencial. Conhecido como O Teorema das Quatro Cores, sua solu c ao resistiu, por mais de 100 anos, ao ataque de muitos matem aticos. Mostrar um pouco dessa hist oria e o objetivo desta nota.

1. Introdu c ao
Nossa hist oria come ca em 1852, quando Francis Guthrie, trabalhando na colora c ao do mapa dos condados da Inglaterra, notou que quatro cores eram sucientes para colorir o mapa, com a condi c ao de que regi oes adjacentes n ao fossem pintadas com a mesma cor. Isso o levou a escrever a seu irm ao Friderick questionando se todo mapa poderia ser colorido utilizando apenas quatro cores. Assim nasceu o problema das quatro cores; Quatro cores s ao sucientes para colorir qualquer mapa no plano com a condi ca o de que regi oes adjacentes (que tenham uma linha de fronteira) n ao sejam coloridas com a mesma cor. Como Friderick n ao conseguiu responder a pergunta de seu irm ao, levou o problema a seu professor De Morgan na University College em Londres. De Morgan observou inicialmente que se um mapa possui quatro regi oes adjacentes duas a duas, ent ao ele necessitar a de quatro cores para ser colorido(veja Fig. 1.1). Procurou ent ao determinar um mapa em que cinco regi oes fossem adjacentes duas a duas. Acabou provando

Olimp ada de Matem atica do Estado de Goi as

78

que tal mapa n ao existe. Infelizmente isso n ao impede que existam mapas que necessitem de cinco cores. De fato, um mapa que n ao possua quatro regi oes adjacentes duas a duas pode necessitar de quatro cores (veja Fig. 1.2). Assim, mesmo n ao tendo o mapa cinco regi oes adjacentes duas a duas pode ele necessitar de cinco cores para ser colorido.

Figura 1.1:

Figura 1.2:

2. A Solu c ao de Kempe
Um pouco mais tarde, em 1878, Arthur Cayley levou esse problema ` a Sociedade de Matem atica de Londres. Em seguida Arthur Kempe apresentou sua solu c ao para o problema. Para acompanhar melhor o que foi feito por Kempe, representaremos um mapa por um conjunto de pontos (v ertices) e arcos (arestas). Dessa forma um mapa e representado por um grafo (conjunto de v ertices e arestas), em que cada regi ao do mapa e representada por um ponto e os arcos ligando dois pontos indicam que essas regi oes s ao adjacentes (vizinhas) (veja Fig. 1.3). Nessa representa c ao, os grafos obtidos a partir de mapas planares s ao tamb em planares, isto e, podem ser desenhados no plano, e a interse c ao de duas arestas, caso exista, e um v ertice.

Figura 1.3: O trabalho de Kempe passou a ser o de colorir v ertices de um grafo, com a condi c ao de que v ertices ligados por uma aresta n ao tenham a mesma cor. Chamando de triangula c ao aquele grafo em que todas as regi oes formadas t em tr es lados (veja Fig. 1.4), Kempe observou que:

Olimp ada de Matem atica do Estado de Goi as

79

1. Todo grafo pode ser estendido a uma triangula c ao, colocando novas arestas. 2. O n umero de cores necess arias para colorir o grafo estendido e suciente para colorir o grafo original.

Figura 1.4:

Figura 1.5:

Com essas observa c oes, passou a preocupar-se apenas com a colora c ao das triangula c oes. Para mostrar que toda triangula c ao pode ser colorida com quatro cores usou o seguinte argumento indutivo. Toda triangula c ao com quatro ou menos v ertices pode ser colorida com quatro cores. Se uma triangula c ao com n v ertices pode ser colorida com quatro cores, ao colocarmos um novo v ertice e o ligarmos aos seus tr es vizinhos, obteremos uma nova triangula c ao com n + 1 v ertices. Esse novo grafo tamb em pode ser colorido com apenas quatro cores, pois o v ertice acrescentado estar a impedido, no m aximo, de usar tr es cores, restando uma para colori-lo. Dessa forma toda triangula c ao obtida a partir de outra pode ser colorida com apenas quatro cores (veja Fig. 1.6).

Figura 1.6: Com esse argumento Kempe acreditou ter resolvido o problema. Infelizmente n ao foi o que aconteceu. Heawood, em 1890, encontrou uma falha na demonstra c ao. Observou que o argumento usado por Kempe

Olimp ada de Matem atica do Estado de Goi as

80

acarretaria na exist encia, em toda triangula c ao, de um v ertice com apenas tr es vizinhos. Com o exemplo ilustrado na gura 1.7, mostrou que isso n ao e verdade. Nem toda triangula c ao pode ser obtida a partir de outra triangula c ao menor. Sem um v ertice que tenha apenas tr es vizinhos a triangula c ao da gura 1.7 n ao pode ser reduzida a uma menor com a retirada de um de seus v ertices e as arestas ligadas a ele. Portanto n ao foi obtido pelo m etodo indutivo apresentado por Kempe.

Figura 1.7:

3. O Probelma das Cinco Cores


Al em de encontrar essa falha no trabalho de Kempe, Heawood provou que cinco cores s ao sucientes para colorir qualquer grafo. Ele denotou por V , A e F o n umero de v ertices, arestas e faces (regi oes limitadas e a regi ao exterior) do grafo. A f ormula de Euler garante que V A + F = 2 para guras formadas por pol gonos justapostos. No caso de uma triangula c ao, temos que cada face tem tr es arestas e cada aresta pertence a duas faces, isto e, 3F = 2A. Substituindo na F ormula de Euler temos que A = 3V 6. Duas conclus oes importantes decorrem dessa f ormula: A primeira e o Teorema de De Morgan, n ao existe mapa com cinco regi oes vizinhas duas a duas. Observe que a f ormula nos diz que um grafo com cinco v ertices ter a, no m aximo, nove arestas, enquanto que seriam exigidas dez para que dois a dois todos os v ertices fossem ligados. A segunda e que a m edia de arestas por v ertices e menor que seis. De fato, como cada aresta chega a dois v ertices, a m edia de arestas por 12 2A ou MA = 6 . Se o grafo n ao e uma v ertices e dada por MA = V V triangula c ao, ent ao essa m edia e menor. Isso nos leva a ` conclus ao de que todo grafo no plano possui pelo menos um v ertice com cinco ou menos

Olimp ada de Matem atica do Estado de Goi as

81

arestas conectadas. Para concluir a demonstra c ao de que cinco cores s ao sucientes para colorir qualquer grafo Heawood usa o seguinte argumento indutivo. No caso em que um dos n v ertices do grafo possui quatro ou menos arestas, removendo esse v ertice e as arestas ligadas a ele obtemos um novo grafo com n 1 v ertices. Se esse grafo menor puder ser colorido com cinco cores, o grafo inicial tamb em poder a, pois o v ertice removido, ao ser recolocado, s o ter a quatro ou menos liga c oes, permitindo que se use uma outra cor para colori-lo. Caso o grafo, com n v ertices, tenha um v ertice v com cinco arestas, dois dos vizinhos de v n ao ser ao vizinhos entre si, conforme De Morgan demonstrou. Fazendo a jun ca o desses dois v ertices a v obtemos um s o v ertice (veja Fig. 1.8). Isso resulta num grafo com n 2 v ertices. Se esse puder ser colorido com cinco cores, o grafo inicial tamb em poder a. Pois, ao separar os dois v ertices de v , podemos manter para esses dois vizinhos de v a cor imposta pela colora c ao. Com isso, o v ertice v estar a impedido, no m aximo, de usar quatro cores, restando uma das cinco para colorir v .

Figura 1.8: Esses dois casos permitem reduzir qualquer grafo a um que tenha apenas cinco v ertices. Portanto cinco cores s ao sucientes para colorir qualquer grafo.

4. O Mapa de Gardner
Muito tempo depois dos resultados obtidos por Heawood, em 1o de Abril de 1975, Martin Gardner elaborou um mapa que, segundo ele, exigiria cinco cores para ser colorido. Mas isso n ao passou de uma brincadeira

Olimp ada de Matem atica do Estado de Goi as

82

no dia da mentira. Deixamos a cargo do leitor a tarefa de colorir o mapa da gura 1.9 usando apenas quatro cores.

Figura 1.9: Al em de colorir o mapa acima, tamb em deixamos para o leitor o problema de determinar, sobre o grafo, condi c oes que garantam a colora c ao com tr es (ou duas) cores.

5. Um Desfecho Inesperado
Mesmo tendo algum avan co, o problema das quatro cores continuava aberto. Para sua demonstra c ao, fazia-se necess ario mostrar que mesmo os grafos n ao contemplados pelo processo desenvolvido por Kempe, como o exemplo apresentado por Heawood, poderiam ser de alguma forma reduzidos sem que isso alterasse o n umero de cores necess arias para sua colora c ao. Em 1977, K. Appel e W. Haken publicaram a demonstra c ao do teorema das quatro cores. Mostraram, com o uso do computador, que toda triangula c ao no plano cont em uma congura c ao especial (dentre 1482 congura c oes especiais) que permite sua colora c ao com quatro cores. No entanto, a parte computacional da demonstra c ao levou mais de 1000 horas para ser processada. Sua extens ao foi alvo de v arias cr ticas e levou ` a busca por uma demonstra c ao mais simples. Em 1994, Robertson, Sanders, Seymour e Thomas reduziram para menos 700 o n umero de congura c oes especiais, mas o computador ainda foi necess ario. Poucos acreditam que exista uma demonstra c ao na qual n ao seja necess ario o uso do computador, mas ainda n ao se descartou essa possibilidade.

Olimp ada de Matem atica do Estado de Goi as

83

Bibliograa
[1] Saaty, T. L. and Kainen, P. C., The Four Color Problem; Assaults and Conquest, Dover Publications, New York, 1986. [2] Courant, R. e Robbins, H. O que e Matem atica?, Editora Ci encia Moderna, Trad. de Adalberto da S. Brito, 2000, RJ. [3] Lines, M., Pense Num N umero, Aprender fazer Ci encia, Gradiva, tradu c ao de Jos e Lu s Malaquias, 1993. [4] Cardoso, D. M., Sobre o Teorema das Quatro Cores. Folha informativa da SPM, 6 (1998):11-14, Dispon vel em: http:www.mat.ua.pt/dcardoso/Papers.htm [5] The Four Color Theorem,2005, Dispon vel em: http:www.mathpages.com/home/kmath266/kmath266.htm [6] Pitombeira, J. B., O problema das liga c oes de agua, luz e telefone: Uma aplica c ao da f ormula de Euler, Revista do Professor de Matem atica, no. 11, 9-16, SBM, 1987. [7] Carneiro, V.C., Colorindo Mapas, Revista do Professor de Matem atica, no. 29, 31-35, SBM, SP, 1995. [8] Carvalho, P. C. P., Dois problemas sobre grafos, EUREKA!, no. 01, 51-57 SBM, RJ, 1998. [9] Lima, E. L., Alguns problemas cl assicos sobre grafos, Revista do Professor de Matem atica, no. 12, 36-42, SBM, 1988. Autor: Endere co: Ronaldo Antonio dos Santos UNIVERSIDADE DE FEDERAL DE GOIAS Campus de Rialma Rua Benedito Luiz Dias, s/n, Setor Alvorada 74.310-000 Rialma, GO, Brasil rasantos@mat.ufg.br

Revista da Olimp ada - IME - UFG, no - 7, Setembro 2008,

84-88

Inteiros Gaussianos e C alculo do Valor de

Lenimar Nunes de Andrade Resumo. Entre as in umeras f ormulas que existem envolvendo a constante mais famosa da Matem atica, t em se destacado algumas f ormulas que envolvem a fun c ao trigonom etrica arco-tangente. Este artigo e sobre a obten c ao de algumas dessas f ormulas e como utiliz alas para obter boas aproxima c oes num ericas dessa constante .

1. Introdu c ao
Desde a antiguidade que o c alculo do valor de tem despertado o interesse de diferentes povos. Aproxima c oes como 3,12 ou 3,16 j a eram conhecidas por babil onios ou eg pcios h a v arios mil enios. Calculado na antiguidade por m etodos puramente geom etricos (inscri c ao e circunscri c ao de pol gonos regulares em uma circunfer encia), a partir do s eculo XVIII passou a ser calculado por m etodos anal ticos, usando-se apenas opera c oes alg ebricas como adi c ao, multiplica c ao e divis ao de n umeros reais. Esses m etodos anal ticos costumam produzir resultados com grande precis ao, ou seja, com muitas casas decimais corretas. Entre os v arios m etodos anal ticos conhecidos, destaca-se uma fam lia de f ormulas que expressam como uma combina c ao de v arios arco-tangentes. Em meados do s eculo XVIII, uma dessas f ormulas foi utilizada para calcular pela primeira vez com 100 casas decimais corretas. A partir do s eculo XX, com a utiliza c ao de computadores cada vez mais potentes e r apidos, o c alculo de passou a ser efetuado com uma quantidade cada vez mais espantosa de casas decimais. Recentemente,

Olimp ada de Matem atica do Estado de Goi as

85

em dezembro de 2002, um recorde foi batido no Jap ao com a ajuda de supercomputadores. Nessa ocasi ao, foi utilizada a f ormula 1 1 1 1 = 44 arctg + 7 arctg 12 arctg + 24 arctg 4 57 239 682 12943 para calcular com mais de um trilh ao e duzentos milh oes de casas decimais! Este artigo relaciona dois assuntos bem diferentes: a fatora c ao de um certo tipo de n umero complexo e f ormulas que envolvem a fun c ao arcotangente que podem ser usadas no c alculo do valor de . As f ormulas mostradas nos exemplos fornecem boas aproxima c oes para usando-se apenas poucas opera c oes aritm eticas.

2. Inteiros gaussianos
Um inteiro gaussiano e um n umero complexo a + bi onde a, b Z. Se z = a + bi, chamamos o produto z z de norma de z e denotamos por N (z ). Portanto, N (a + bi) = a2 + b2 . Podemos ver de imediato que N (z1 z2 ) = N (z1 )N (z2 ). Os inteiros gaussianos possuem algumas propriedades alg ebricas muito parecidas com as dos inteiros. Alguns inteiros gaussianos podem ser escritos como produto de dois outros inteiros gaussianos de normas maiores do que 1 (por exemplo, 12 + i = (2 + 5i)(1 2i)), enquanto que outros n ao admitem tal fatora c ao (por exemplo, 1 i), comportando-se de modo semelhante aos inteiros primos. Para fatorar z = a + bi, a, b Z, b = 0, nos inteiros gaussianos, primeiramente calculamos N (z ). Se N (z ) for primo, ent ao z n ao se fatora como produto de dois inteiros gaussianos de normas maiores do que 1. Se 2 for um divisor de N (z ), ent ao 1 + i e um divisor de z . Se p = 4k + 1, k N, for um divisor primo de N (z ), ent ao existem a, b Z tais que p = a2 + b2 ; neste caso, a + bi ou b + ai e divisor de z . Se p = 4k + 3, k N, for um divisor primo de N (z ), ent ao p tamb em e um divisor de z .

Olimp ada de Matem atica do Estado de Goi as

86

A demonstra c ao deste resultado pode ser encontrada em [3]. Exemplo 1. Vamos fatorar z1 = 5 + i e z2 = 239 i no conjunto dos inteiros gaussianos. Como N (z1 ) = 52 + 12 = 26 = 2 13, temos que 1 + i e um fator de z1 . Dividindo z1 por 1 + i obtemos 3 2i. Logo, z1 = (1 + i)(3 2i). Temos tamb em N (z2 ) = 2392 + (1)2 = 57122 = 2 134 z2 se fatora como produto de inteiros gaussianos de norma 2 (que e 1 + i) e de norma 13 (que podem ser 3 + 2i ou 2 + 3i). Por tentativas, obtemos seguinte fatora ca o para z2 : z2 = (1 + i)(3 + 2i)4 . A fatora c ao de inteiros gaussianos pode ser bastante trabalhosa. No entanto, os programas de Computa c ao Alg ebrica (como o Maple), bastante comuns hoje em dia, fazem essa fatora c ao na maior rapidez.

3. F ormulas envolvendo a fun c ao arco-tangente


Todo n umero complexo z = x + yi pode ser escrito na forma trigonom etrica z = (cos + i sen ). Neste caso, e a raiz quadrada positiva de N (z ) e , chamado argumento de z e denotado por arg(z ), possui as seguintes propriedades: arg(z ) = arctg
y x

+ k , x = 0, k Z;

arg(z1 z2 ) = arg(z1 ) + arg(z2 ) + 2k , k Z; arg(z n ) = n arg(z ) + 2k , k Z, n N. Em todos os exemplos a seguir, tivemos o cuidado de usar apenas casos em que k = 0 nas f ormulas que envolvem c alculos de argumentos. Exemplo 2. Neste exemplo, vamos mostrar que = 16 arctg 1 1 4 arctg . 5 239

y , consideramos uma pot encia Para cada parcela do tipo n arctg x 1 1 n de inteiro gaussiano da forma (x + iy ) . Como arctg 239 = arctg 239 , 16 4 vamos inicialmente avaliar o produto (5 + i) (239 i) .

Olimp ada de Matem atica do Estado de Goi as

87

Fatorando 5 + i e 239 i, temos 5 + i = (1 + i)(3 2i), isto e, 239 i = (1 + i)(3 + 2i)4 e da : (5 + i)16 (239 i)4 = (1 + i)20 (3 2i)16 (3 + 2i)16 = (2i)10 (13)16 = 210 1316 . = ((1 + i)2 )10 ((3 2i)(3 + 2i))16

1 O argumento de z = (5 + i)16 (239 i)4 e 16 arctg 1 5 + 4 arctg 239 . Por outro lado, como z e um n umero real negativo, ele tem argumento igual a . Obtemos assim a igualdade desejada. Essa f ormula foi utilizada no s eculo XVIII para calcular com 100 casas decimais.

Exemplo 3. Vamos obter agora outra f ormula envolvendo e a fun ca o arco-tangente. Um procedimento an alogo pode ser utilizado para se obter muitas outras f ormulas semelhantes. Inicialmente, escolhemos dois n umeros complexos z1 e z2 e inteiros m z n tenha um argumento f m, n N tais que z1 acil de ser calculado 2 (como uma pot encia de 1 + i ou de i, por exemplo). Dessa forma, escolhemos aleatoriamente z1 = (1 + i)(1 + 2i)5 = m z n , devemos 79 + 3i. Para que 1 + 2i n ao compare ca no produto z1 2 escolher um z2 que contenha um fator que seja pot encia de 1 2i, por exemplo z2 = (1 + i)(1 2i)12 = 22049 + 1457i. 12 z 5 )4 obtemos: z 48 z 20 = ((1 + i)17 (1 + Calculando o produto (z1 1 2 2 60 60 4 4 17 2i) (1 2i) ) = ((1+ i) ) ((1+2i)(1 2i))240 = 417 5240 . O expoente 4 que aparece nesta express ao e opcional; sem ele obter amos o mesmo resultado. 3 48 z 20 = (79+3i)48 (22049+1457i)20 + O argumento de z1 e 48 arctg 79 2 1457 e um n umero real negativo devemos ter tamb em 20 arctg 22049 . Como ele argumento igual a . Portanto, obtivemos a seguinte igualdade: = 48 arctg 3 1457 + 20 arctg . 79 22049

Exemplo 4. Se dispus essemos de uma calculadora com apenas as opera co es aritm eticas b asicas, seria poss vel vericarmos a validade de f ormulas como a do exemplo anterior? A resposta e sim. Se |x| < 1, ent ao o arco-tangente de x pode ser aproximado por um polin omio formado por pot encias mpares de x, com sinais alternados, tendo-se o cuidado de

Olimp ada de Matem atica do Estado de Goi as

88

dividir cada pot encia pelo seu respectivo expoente. Em outras palavras, arctg x pode ser aproximado pelo polin omio Pn (x) = x x2n1 x3 x5 x7 + + + (1)n+1 . 3 5 7 2n 1

Quanto maior o n e mais pr oximo de 0 for x, melhor ser a a aproxima ca o. 3 x5 x7 Escolhendo P4 (x) = x x ca o do arctg x, 3 + 5 7 como aproxima usando o exemplo anterior temos: 48 arctg 1457 3 1457 3 + 20 arctg 48P4 ( ) + 20P4 ( ) 79 22049 79 22049 48P4 (0, 0379746835) + 20P4 (0, 0660800943) 48 0, 0379564451 + 20 0, 0659841642 3, 1415926535.

Assim, com poucas opera co es aritm eticas obtivemos o valor de com 10 casas decimais exatas. A f ormula utilizada no recorde do c alculo do valor de em 2002 tamb em pode ser obtida e demonstrada por um procedimento semelhante. Por m, deixamos proposto o seguinte exerc cio: 1 1 1 = 5 arctg + 2 arctg + 3 arctg . 4 8 18 57 Sugest ao: 8 + i = (3 + 2i)(2 i), 18 + i = (3 2i)(2 i)2 i, 57 + i = (3 2i)(2 + i)3 (1 i). Exerc cio 1. Mostre que

Bibliograa
[1] M. Beeler, R. W. Gosper, R. Schroeppel, MIT AI Memo 239, Feb. 29, 1972. [2] V. Bongiovanni, R. Watanabe, Pi acaba?, Revista do Professor de Matem atica 19, 1991, pp. 17. [3] A. Hefez, Curso de Algebra, vol. 1 Cole c ao Matem atica Universit aria, 1993.

Olimp ada de Matem atica do Estado de Goi as

89

Autor: Endere co:

Lenimar Nunes de Andrade UNIVERSIDADE DE FEDERAL DA PARA IBA Departamento de Matem atica Cidade Universit aria - Campus I 58.051-900, Jo ao Pessoa, PB, Brasil lenimar@mat.ufpb.br

Revista da Olimp ada - IME - UFG, no - 7, Setembro 2008,

90-94

Calculando Logaritmos de uma Forma Eciente

Lenimar Nunes de Andrade Resumo. Assim como existem f ormulas com a fun c ao trigonom etrica arco-tangente que s ao usadas no c alculo do valor de , existem f ormulas que envolvem a fun c ao arco-tangente hiperb olica que s ao u teis no c alculo de logaritmos. Neste artigo, destacamos a utiliza c ao dessas f ormulas no c alculo de logaritmos de alguns n umeros.

1. Introdu c ao
Desde o s eculo XVII que os logaritmos v em sendo utilizados. Seu c alculo despertou a aten c ao de matem aticos famosos como Newton, Euler, entre outros. Os valores dos logaritmos de v arios n umeros eram publicados em forma de longas tabelas chamadas t abuas de logaritmos. Devido as propriedades dos logaritmos, a utiliza c ao dessas tabelas tinha por objetivo facilitar c alculos onde apareciam multiplica c oes, divis oes, pot encias e ra zes. Como as t abuas de logaritmos s ao constru das e a maneira como as calculadoras ou computadores os calculam e algo que sempre chama a aten c ao dos curiosos. Neste artigo apresentamos algumas f ormulas que podem ser usadas para calcular logaritmos de uma forma eciente, ou seja, com poucas opera c oes aritm eticas envolvidas e boa precis ao num erica dos resultados obtidos.

2. A fun c ao Arco-tangente Hiperb olica


A fun c ao arco-tangente hiperb olica arctgh(x) pode ser denida por arctgh(x) = 1 ln 2 1+x 1x ,

Olimp ada de Matem atica do Estado de Goi as

91

onde ln(x) = loge (x) representa o logaritmo natural de x. Em geral, para toda f ormula envolvendo uma fun c ao trigonom etrica existe uma f ormula an aloga envolvendo uma fun c ao hiperb olica. Por exemplo, a partir das conhecidas f ormulas para o c alculo da soma dos termos de uma progress ao geom etrica innita com |x| < 1 1 = 1 x2 + x4 x6 + x8 1 + x2 e 1 = 1 + x2 + x4 + x6 + x8 + 1 x2 podemos obter as f ormulas arctg x = e arctgh x =
k =0

(1)k x2k+1 x3 x5 x7 x9 = x + + 2k + 1 3 5 7 9

k =0

x2k+1 x3 x5 x7 x9 = x+ + + + + . 2k + 1 3 5 7 9

Uma f ormula muito utilizada para calcular o valor de e a f ormula de Machin: 1 1 = 4 arctg arctg . 4 5 239 Usando essa f ormula, ele calculou com 100 casas decimais em 1706. Temos v arias f ormulas parecidas com a f ormula de Machin onde aparecem logaritmos no lugar de . Como por exemplo, 1 1 ln 2 = 2 arctgh + 2 arctgh , 5 7 (que e equivalente a ln 2 = ln(3/2) + ln(4/3)). Essa f ormula foi usada imprespor Euler em 1748 para calcular ln 2 com 25 casas decimais. E sionante o fato de n umeros t ao distintos quanto e ln 2 serem obtidos atrav es de c alculos t ao semelhantes. Muitas outras f ormulas podem ser obtidas a partir de valores particulares das fun c oes arctgh(x) e ln(x). Por exemplo, a partir de 1/2 ln(3/4) + arctgh(1/2) pode-se chegar a ln 2 =
k =0

1 1 + 8k + 8 4k + 2

1 , 4k

que e uma s erie de converg encia muito r apida, somando-se poucos termos podemos obter resultados bem pr oximos do valor exato.

Olimp ada de Matem atica do Estado de Goi as

92

3. Uma F ormula Eciente


Em 1997, P. Sebah obteve a f ormula 13 1 + 4 arctgh . 17 499 A verica c ao da validade desse tipo de f ormula em geral e imediata, bastando usar as deni c oes e propriedades b asicas das fun c oes envolvidas. Por exemplo, a f ormula anterior e equivalente a ln 2 = 10 arctgh ln 2 = 5 ln 9 8 + 2 ln 256 243 = 5(2 ln 3 3 ln 2) + 2(8 ln 2 5 ln 3).

Essa f ormula foi utilizada em 2001 para calcular ln 2 com mais de 500 milh oes de casas decimais. Substituindo-se arctgh(1/17) e arctgh(13/499) pela soma de tr es termos de cada uma das respectivas s eries, podemos obter um valor aproximado de ln 2 com 8 casas decimais exatas (isto e, 8 casas decimais do valor aproximado coincidindo com as do valor exato), conforme mostrado a seguir. Utilizamos o s mbolo signicando aproximadamente igual a. 1 1 1 1 + + = 0, 05889152, arctgh 17 17 14739 7099285 13 13 2197 371293 arctgh + + = 0, 02605800, 499 499 372754497 154693737512495 ln 2 10(0, 05889152) + 4(0, 02605800) = 0, 69314718.

4. Logaritmos de Outros N umeros


Conhecendo-se um valor como o de ln 2, usando propriedades b asicas dos logaritmos podemos determinar v arios outros como ln 0, 5 = ln(1/2) = ln 2, ln 4 = 2 ln 2, ln 0, 25 = ln(1/4) = 2 ln 2, etc. Al em disso, a partir de 2 arctgh 1 2x + 1 = ln 1+ 1
1 2x+1 1 2x+1 1 2x+1

= ln

x+1 x

obtemos ln(x + 1) = ln(x) + 2 arctgh ln(x + 1) = ln(x) + 2

, ou seja,

1 1 1 + + + , 2x + 1 3(2x + 1)3 5(2x + 1)5

Olimp ada de Matem atica do Estado de Goi as

93

que pode ser usada para calcular logaritmos de outros n umeros, conforme mostrado a seguir onde a partir de ln 2 calculamos ln 3, depois ln 9 e ln 10. 1 1 1 1 1 + + + + = 1, 09861228, ln 3 ln 2+2 5 375 15625 546875 17578125 ln 9 = ln 32 = 2 ln 3 = 2 1, 09861228 = 2, 19722456 e ln 10 ln 9 + 2 1 1 1 1 + + + + 19 20577 12380495 6257102173 1 + = 2, 30258508. 2904189280011

5. Logaritmos Decimais
Para obter logaritmos decimais, basta dividir os logaritmos naturais por ln 10. Podemos obter assim os seguintes valores: 0, 69314718 ln 2 = = 0, 30103000, ln 10 2, 30258508 ln 3 1, 09861228 log 3 = = = 0, 47712125 ln 10 2, 30258508 log 2 = que s ao valores com 8 casas decimais exatas. Note que usamos apenas uma quantidade bem modesta de termos dos desenvolvimentos em s eries de pot encias. Se tiv essemos usado mais termos, ter amos obtidos resultados muito melhores. Por exemplo, se tiv essemos usado 10 termos dos desenvolvimentos de cada s erie, no nal ter amos obtido log 2 e log 3 com 15 casas decimais exatas.

6. Recordes no C alculo de ln 2
Em 2001, foi divulgado um c alculo de ln 2 com mais de 500 milh oes de casas decimais. A seguir, a evolu c ao da quantidade de casas decimais desse tipo de c alculo ao longo de v arias d ecadas.

Olimp ada de Matem atica do Estado de Goi as

94

N. d gitos 16 25 137 273 330 3.683 2.015.926 5.039.926 10.079.926 29.243.200 58.484.499 108.000.000 200.001.000 240.000.000 500.000.999

Ano 1671 1748 1853 1878 1940 1962 1997 1997 1997 1997 1997 1998 2001 2001 2001

Calculador I. Newton L. Euler W. Shanks Adams H. S. Uhler D.W. Sweeney P. Demichel P. Demichel P. Demichel X. Gourdon X. Gourdon X. Gourdon X. Gourdon & S. Kondo X. Gourdon & P. Sebah X. Gourdon & S. Kondo

Bibliograa
[1] Avila, G., Como se constr oi uma t abua de logaritmos, Revista do Professor de Matem atica 26, 1994. [2] Gourdon, X., Sebah, P., The logarithm constant log(2), 2001, dispon vel na internet em numbers.computation.free.fr [3] Markushevich, A. I., Areas y logaritmos, Editorial Mir, 1975. Autor: Endere co: Lenimar Nunes de Andrade UNIVERSIDADE DE FEDERAL DA PARA IBA Departamento de Matem atica Cidade Universit aria - Campus I 58.051-900, Jo ao Pessoa, PB, Brasil lenimar@mat.ufpb.br

Revista da Olimp ada - IME - UFG, no - 7, Setembro 2008,

95-98

Progress oes - Uma Atividade de Introdu c ao ao Conceito de Limite

Eudes Antonio da Costa Resumo. Alguns Paradoxos t em desaado e estimulado a raz ao humana ao logo dos tempos, entre eles, o Paradoxo de Zen ao. Nossa tentativa ser a analisar (formular) este paradoxo matematicamente. Usaremos a geometria euclidiana e as progress oes para introduzir o conceito, intuitivo, de limite. Palavras-chave: Movimento, Paradoxo e Limite.

1. Sobre o Movimento
Zen ao de El eia viveu na Gr ecia antiga, no s eculo V a.C. Zen ao elaborou quatro aporias (argumentos), nestes pretendia mostrar que o movimento n ao podia ser racionalmente explicado [6]. A aporia que vamos analisar e conhecida como dicotomia e nos conduz a um paradoxo. Por paradoxo [1] entendemos aquilo que n ao tem solu c ao, algo confuso, algo contr ario ao nosso bom-senso, contr ario aos (nossos) conhecimentos anteriores teses, certezas. Vejamos a aporia da dicotomia (adaptado)[4]: Um atirador est a a certa dist ancia de um alvo, admita que a echa percorra sempre a metade do caminho, entre ela e o ponto a ser atingido. Pergunta: Seria poss vel a echa, ao ser lan cada, alcan car um alvo disposto a alguma dist ancia do atirador? O que Zen ao pretendia, com suas aporias, n ao era negar o movimento aparente da echa at e o alvo, n ao se est a em discuss ao o eu vejo, pois o atirador ao lan car a echa, com for ca suciente, eu vejo a echa atingir o alvo.

Olimp ada de Matem atica do Estado de Goi as

96

O que Zen ao pretendia era derrubar a tese pitag orica de que a matem atica (geometria) podia explicar todos os fatos e a tese de Her aclito de que o movimento era a u nica realidade [2]. Zen ao pretende mostrar que o movimento e racionalmente absurdo, da instala o paradoxo, pois o movimento faz parte do nosso bom-senso, do nosso sentir, do nosso ver. Mas imposs vel de ser explicado, racionalmente (naquela epoca).

2. Representa c ao Matem atica

X1

Sejam A o lugar (ponto) em que o atirador se encontra e B o ponto em que se encontra o alvo, de coordenadas a e b [3], respectivamente. Admitindo que a echa para percorrer de A at e B tem que primeiro passar por um ponto X1 que esta ` a metade da distancia entre A e B , isto e, X1 e o ponto m edio do segmento AB e possui coordenada x1 = a+b ab 2 = a 2 . Aplicando novamente o argumento, saindo de A em dire c ao ` a X1 , temos que existe um ponto X2 que e ponto m edio do a+x1 ab segmento AX1 , sendo x2 = 2 = a 22 . Aplicando indenidamente o argumento constru mos a seq u encia: a+b ab =a ; 2 2 a + x1 ab x2 = =a 2 ; 2 2 a + x2 ab x3 = =a 3 ; 2 2 ... ab a + xn =a n , xn = 2 2 x1 =

para n 1.

Como 2n cresce (muito) r apido, ` a medida que n aumenta. (Para ver isso, compare os termos da seq u encia (progress ao geom etrica) (2n ) = (2, 4, 8, 16, 32, 64, 128, 256, 512, 1024, . . . ).

Olimp ada de Matem atica do Estado de Goi as

97

Isto e, xn e seq u encia decrescente [5]. Pois xn < xn1 j a que 2n > 2n1 ab e pr oximo de para todo n 1. Ou seja, quanto maior o n tanto 2n zero. Assim temos que xn aproxima de a.

3. Primeira Considera c ao
Surpreendentemente admitindo que, em um movimento entre um ponto inicial e um ponto nal, primeiro (sempre) atinge-se um ponto intermedi ario, leva-nos a um absurdo que n ao podemos sair do lugar (Paradoxo). Novamente n ao se est a em jogo o eu vejo, mas como explicar racionalmente (matematicamente) o movimento admitindo estas premissas (pitag orica): a matem atica pode explicar todos os fatos e o movimento e uma realidade.

4. Segunda Considera c ao (Outra Tentativa)

X1

Admitiremos agora que a echa saindo de A at e B necessariamente passa por um ponto X1 que esta ` a metade da distancia entre A e B , isto e, X1 e o ponto m edio do segmento AB . Ou seja, atingir X1 e uma b ab realidade, tomemos x1 = a+ = b + . 2 2 Estando agora em X1 , e desejando (ainda) atingir B , aplica-se novamente o argumento, saindo de X1 em dire c ao ` a B , temos que existe um ponto X2 que e ponto m edio do segmento X1 B sendo x2 = x 1 +b ab mos a 2 = b + 22 . Aplicando indenidamente o argumento constru seq u encia: a+b ab =b+ ; 2 2 x1 + b ab x2 = =b+ 2 ; 2 2 x2 + b ab x3 = =b+ 3 ; 2 2 ... xn + b ab xn = =b+ n , 2 2 x1 =

para n 1.

Olimp ada de Matem atica do Estado de Goi as

98

Agora temos que temos que xn aproxima de b.

5. Ultima Considera c ao
N ao pretendemos neste discutir losocamente, nem historicamente as aporias de Zen ao [2]. Apenas ilustrar uma atividade de no c ao intuitiva de limite. Percebemos que conforme entendemos o conceito de movimento atinge-se o ponto nal ou chega-se a impossibilidade do movimento.

Bibliograa
[1] ABBAGNANO, Nicola. Dicion ario de Filosoa. Fundo de Cultura Econ omica, M exico, 1963. [2] BARKER, Stephen F. Filosoa da Matem atica. Zahar Editores: Rio de Janeiro. 1969. [3] BARBOSA, Jo ao Lucas Marques. Geometria Euclideana Plana. Cole c ao Professor de Matem atica, SBM, 1995. [4] BOYER, Carl B. Hist oria da Matem atica. Edgard Bl ucher Ltda: S ao Paulo. 1991. [5] LEITHOLD, Louis. O C alculo com Geometria Anal tica. Editora Harbra ltda: S ao Paulo. 1994. [6] Os Pr e-Socr aticos - Os Pensadores. 2a ed. S ao Paulo: Abril Cultural, 1976. Autor: Endere co: Eudes Antonio da Costa UNIVERSIDADE DE FEDERAL DO TOCANTINS Campus Universit ario de Arraias Curso de Matem atica Av. Universit aria, s/n, Centro 77.330-000 Arraias, TO, Brasil eudes@uft.edu.br

Revista da Olimp ada - IME - UFG, no - 7, Setembro 2008,

99-111

N umeros Perfeitos e Primos de Mersenne

Alacyr J. Gomes, Eudes A. da Costa e Ronaldo A. dos Santos Resumo. Nesta nota apresentaremos algumas propriedades e rela c oes envolvendo n umeros perfeitos e primos de Mersenne.

1. Introdu c ao
Os n umeros perfeitos fascinam os matem aticos a s eculos. Acredita-se que Pit agoras esteja entre os primeiros a se deslumbrarem com tais n umeros. Mas quais s ao e que mist erios cercam os n umeros perfeitos ? A deni c ao, encontrada nos Elementos de Euclides, diz que; N umero perfeito e aquele que e igual a soma de suas partes, entendendo como parte de um n umero seus divisores pr oprios. Os quatro1 primeiros n umeros perfeitos s ao: 6 = 1 + 2 + 3; 28 = 1 + 2 + 4 + 7 + 14; 496 = 1 + 2 + 4 + 8 + 16 + 31 + 62 + 124 + 248; 8128 = 1 + 2 + 4 + 8 + 16 + 32 + 64 + 127 + 254 + 508 + 1016 + 2032 + 4064. Que eram os n umeros perfeitos conhecidos at e o s eculo XVI (ver [7]). Mas por que chamar um n umero com tal propriedade de perfeito ? A resposta a essa pergunta e ainda mais curiosa, acredita-se que o nome esteja ligado ao fato de os antigos escribas observarem que seis foram os dias necess arios para a Cria c ao do Mundo, com toda sua perfei c ao2 .
Veja em www.research.att.com/njas/sequences/A000396 uma lista com outros n umeros perfeitos 2 God created the world in 6 days, a perfect number. The moon circles the earth in 28 days, again a symbol of perfection in the best of all possible worlds.(Ore, 1988: 91)
1

Olimp ada de Matem atica do Estado de Goi as

100

Esse tipo de observa c ao serviu para envolver esses n umeros em uma esp ecie de aura, motivando ainda mais seu estudo. Apesar desse lado meio m stico, a verica c ao de que um n umero e perfeito ou n ao e bastante simples, basta somar seus divisores pr oprios. Conforme a deni c ao, se tal soma for igual ao pr oprio n umero ele ser a perfeito. Diz-se ainda que um n umero e abundante se a soma dos divisores pr oprios for maior que o pr oprio n umero e que e deciente se for menor.

2. A Fun c ao e Algumas Propriedades


Euler3 chamou de (sigma) a fun c ao que associa a cada natural n a soma de seus divisores, isto e, (n) e a soma dos divisores positivos de n, incluindo o n. Vejamos alguns exemplos; A soma dos divisores de 17 e 18, pois os divisores de 17 s ao 1 e 17. De modo geral, se p e primo a soma de seus divisores e p + 1. Na terminologia de Euler esse fato se resume em; (p) = p + 1 para todo p primo. A soma dos divisores de 27 e 40, pois os divisores de 27 s ao 1, 3, 3 0 1 2 9 e 27. Observe que 27 = 3 , seus divisores s ao 3 , 3 , 3 e 33 e que 0 1 2 3 40 = 3 + 3 + 3 + 3 . De modo geral, se p e primo e n N, a soma dos divisores de pn e dada por 1 + p + p2 + + pn que sendo a soma dos termos de uma n+1 1 n+1 1 , assim temos que (pn ) = p p1 , progress ao geom etrica e igual a p p1 para p primo e n N. Outro exemplo bastante instrutivo e calcular a soma dos divisores 3 de 200. Como a fatora c ao de 200 e 2 .52 seus divisores s ao da forma, 0 0 0 1 0 2 1 0 1 1 1 2 2 .5 , 2 .5 , 2 .5 , 2 .5 ,2 .5 ,2 .5 ,...,23 .52 . Com isso temos que, (200) = 20 .50 + 20 .51 + 20 .52 + 21 .50 + 21 .51 + 21 .52 + ... + 23 .52 = 20 .(50 + 51 + 52 ) + 21 (50 + 51 + 52 ) + 22 (50 + 51 + 52 ) + 23 (50 + 51 + 52 ) = (20 + 21 + 22 + 23 )(50 + 51 + 52 ) = 15.31 = 465.
3

Leonard Euler (1707-1783), matem atico su co.

Olimp ada de Matem atica do Estado de Goi as

101

Com essa mesma id eia, se p e q s ao primos distintos e n, m N, pode-se vericar que (pn q m ) = (1+p+p2 + +pn )(1+q +q 2 + +q m ) = pn+1 1 q m+1 1 . p1 q1

De modo mais geral, se a decomposi c ao de um n umero natural x e k 1 2 3 ao; dada por p1 p2 p3 pk , ent (x) =
1 +1 p 1 1 p1 1 2 +1 p 1 2 p2 1 3 +1 p 1 3 p3 1

k +1 p 1 k . pk 1

Com essa fun c ao os n umeros perfeitos s ao caracterizados por; n e perfeito se, e somente se, (n) = 2n. J a que a fun c ao soma tamb em n como um dos divisores. Arma c ao 1. Seja n N . Tem-se que (n) = 1 + n se, e s o se, n e um n umero primo. Demonstra ca o. Se (n) = 1 + n, segue que n > 1 e que os divisores de n s ao 1 e n; logo, n e primo. Reciprocamente, se n e primo, os os divisores de n s ao 1 e n, ent ao (n) = 1 + n. Arma c ao 2. Sejam m, n (m n) = (m) (n). N com mdc(m, n) = 1 ent ao

Demonstra ca o. Temos que mdc(m, n) = 1, assim m e n n ao possuem divisores comuns al em do 1. Considere Dm = {1 = x0 , x1 , x2 , . . . , xk = m} e Dn = {1 = y0 , y1 , y2 , . . . , ys = n} o conjunto de divisores de m e n respectivamente, com xi = yj para todo i, j 1. Assim, um n umero ser a divisor de m.n se, e somente se, for da forma xi yj para algum xi Dm e yj Dn , assim
k s k s

(mn) =
d|mn s

d=
i=0 j =0 k

xi y j =
i=0

xi
j =0

yj

=
j =0

yj
i=0

xi = (m) (n).

Olimp ada de Matem atica do Estado de Goi as

102

3. Fatora c oes que n ao Geram N umeros Perfeitos


Arma c ao 3. Um quadrado perfeito n ao e um n umero perfeito. Demonstra ca o. Seja s2 nosso quadrado perfeito. Suponha que s se dek 2 2 1 componha da forma s = p 1 p2 . . . pk . A soma dos divisores de s ser a dada por,
21 22 2k 2 2 (s2 ) = (1+p1 +p2 1 + + p1 )(1+ p2 + p2 + + p2 ) (1+ pk + pk + + pk ).

Como cada um dos fatores acima e n umeros mpar, temos que (s2 ) 2 2 e mpar. Portando (s ) = 2s . Arma c ao 4. N ao existe n umeros perfeitos cuja decomposi ca o e da forma pm q n com 2 < p < q , sendo p, q primos. Demonstra ca o. Para que o n umero acima seja perfeito devemos ter que, (pm q n ) = 2pm q n pm+1 1 q n+1 1 = 2pm q n p1 q1 pqpm q n pm+1 q n+1 + 1 = 2(p 1)(q 1)pn q m

pm+1 q n+1 + 1 = (2(p 1)(q 1) pq )pm q n .

Para que a identidade se verique devemos ter, pq 2p 2q + 2 < 0. O que n ao ocorre, pois, pq 2p 2q + 2 = p(q 2) 2(q 1)

> p(q 2) 2((q 1) 1) = (p 2)(q 2) > 0.

Arma c ao 5. N ao existe n umeros perfeitos cuja decomposi ca o e da m n s forma p q r com 5 p < q < r . Demonstra ca o. Seguindo o mesmo caminho do caso anterior. Supomos que pm q n r s seja um n umero perfeito, isto e, (pm q n r s ) = 2pm q n r s .

Olimp ada de Matem atica do Estado de Goi as

103

Isso implica em 2(p 1)(q 1)(r 1) pqr < 0 1 (p 1) (q 1) (r 1) < . p q r 2 Mas a desigualdade acima nunca ocorre, pois, (p 1) (q 1) (r 1) > p q r p1 p
3

ou

51 5

1 > . 2

Generalizando a id eia acima, temos o seguinte resultado. Arma c ao 6. N ao existe n umero perfeito cuja decomposi ca o seja k 2 k 1 2 p1 < p2 < < pk . p 1 p2 . . . pk , com k 21
1 +1 2 +1 pk +1 1 p 1 p 1 k 1 1 2 2 ... k = 2p 1 p2 . . . pk p1 1 p2 1 pk 1

Demonstra ca o. Para que o n umero acima seja perfeito devemos ter,


k 1 2 (p 1 p2 . . . pk ) =

ou seja

k 1 +1 2 +1 1 2 k +1 (p 1)(p 1) . . . (p 1) = 2(p1 1)(p2 1) . . . (pk 1)p 1 2 1 p2 . . . pk . k


k +1 1 +1 2 +1 Subtraindo em ambos os membros acima o n umero p p2 . . . p , 1 k temos que o primeiro membro e um n umero negativo, conseq uentemente

2(p1 1)(p2 1) . . . (pk 1) p1 p2 . . . pk < 0 ou seja pk 1 1 p1 1 p2 1 < . p1 p2 pk 2 Mas isso n ao ocorre, j a que,
k k 2 k 2 1 k 2 k 21

p1 1 p2 1 pk 1 > p1 p2 pk

p1 1 p1

1 . 2

Olimp ada de Matem atica do Estado de Goi as

104

4. N umeros Perfeitos e Primos de Mersenne


Al em da deni c ao de n umeros perfeitos os Elementos de Euclides (proposi c ao 36 do livro IX) tamb em continham a demonstra c ao de que se o n 4 n 1 n n umero 2 1 e primo , ent ao 2 (2 1) e perfeito; n ao determinou outros n umeros perfeitos al em dos quatro primeiros por causa da diculdade de obter novos n umeros primos da forma 2n 1. Na epoca ou nico processo conhecido para vericar que um dado n umero inteiro a>1 e primo, era o processo das divis oes de a por todos os n umeros ostenes. At e hoje primos p tais que 2 p a, chamado crivo de Erat n ao foi encontrado qualquer n umero perfeito fora do modelo dado por Euclides h a mais de 2.300 anos.5 Exerc cio 1. Mostre que se 2n 1 e primo, ent ao n e primo. Exerc cio 2. D e um exemplo para mostrar que a rec proca, do exerc cio anterior, n ao e verdadeira. O resultado abaixo, parte devida de Euclides e parte devida a Euler, caracterizar a os n umeros perfeitos pares, relacionando-os com os primos de Mersenne. Arma c ao 7. (Euclides - Euler) Um n umero natural n e um n umero perfeito par se, e s o se, n = 2p1 (2p 1) sendo 2p 1 um primo. Demonstra ca o. Suponha que n = 2p1 (2p 1), sendo 2p 1 primo, logo p > 1, e conseq uentemente, n e par. Como 2p 1 e mpar, temos que p 1 p mdc(2 , 2 1) = 1; assim (n) = (2p1 (2p 1)) = (2p1 ) (2p 1) = 2p 1 p 2 = 2n. 21

Portanto, n e perfeito. Reciprocamente, suponha que n e perfeito e par. Seja 2p1 a maior pot encia de 2 que divide n. Logo, p > 1 e n = 2p1 q com q mpar. Ent ao p p p (n) = (2 1) (q ). Como (n) = 2n segue que (2 1) (q ) = 2 q . Da
Primos da forma 2n 1 s ao chamados primos de Mersenne (Marin Mersenne (1588-1648), matem atico franc es). 5 No s eculo XVIII Euler provou que todos os n umeros perfeitos pares t em aquela forma. Em 2004 Davis (matem atico australiano), prop os uma prova de que n ao existem n umeros perfeitos mpares[3].
4

Olimp ada de Matem atica do Estado de Goi as

105

temos que (2p 1)|q pois mdc(2p , 2p 1) = 1. Logo existe c N com c < q tal que q = c(2p 1); portanto (2p 1) (q ) = 2p q = 2p (2p 1)c, logo (q ) = 2p c. Temos que q e c s ao dois divisores distintos de q tais que c + q = 2p c. Nessa situa c ao, c = 1. De fato, suponha, que c = 1. Temos, ent ao, que (q ) 1 + c + q > c + q = 2p c; disto segue que 2p c = c + q < (q ) = 2p c, uma contradi c ao. Portanto, temos que (q ) = q + 1, ou seja q e primo. p 1 p p Temos assim que n = 2 (2 1), sendo 2 1 e primo. Com esse resultado para cada primo de Mersenne teremos um n umero perfeito par. At e o momento j a foram encontrados 44 primos de Mersenne, sendo que o u ltimo6 , p = 232582657 1, com 9.808.358 d gitos. Observe que se cada um dos d gitos desse n umero primo tivesse 1 cm, ele pr oprio teria quase 100 km. Incr vel! Mas mesmo tendo encontrado primos de Mersenne grandes ainda n ao se sabe demonstrar que existem innitos primos de Mersenne nem que existem innitos primos p para os quais Mp = 2p 1 e composto. Tomando p = 232582657 1 o maior primo de Mersenne conhecido, temos que q = 232582656 p e o maior n umero perfeito conhecido. Exerc cio 3. Mostre que todo n umero perfeito N par e triangular e que 1 e tamb em triangular7 . se N > 6 tem-se que N 9 Arma c ao 8. Se n = 2p1 (2p 1) e um n umero perfeito, ent ao o p produto de seus divisores en . Demonstra ca o. Denotaremos por P = 2p 1 o primo de Mersenne. Neste caso os divisores de n s ao do tipo 2i P j , com 0 i p 1 e 0 j 1. Assim o produto dos divisores de n ser a
20 P 0 21 P 0 2p1 P 0 20 P 1 21 P 1 2p1 P 1 = 21+2++(p1) .21+2++(p1) .P p = 2 2 + 2 Pp = 2p(p1) P p = (2p1 P )p = np .
p(p1) p(p1)

Arma c ao 9. A soma dos rec procos dos divisores de um n umero perfeito e 2.


Ou ltimo n umero primo de Mersenne foi descoberto em 2006.[Consulte na Internet a p agina http://primes.utm.edu] +1) 7 , com n natural, (veja [7]) N umeros triangulares s ao aqueles da forma n(n 2
6

Olimp ada de Matem atica do Estado de Goi as

106

Demonstra ca o. Seja n o n umero perfeito e a1 , a2 , a3 , . . . , ar seus divisores. n n n n } , , , ..., a Observe que os conjuntos {a1 , a2 , a3 , . . . , ar } e { a r 1 a2 a3 s ao iguais, isto e, para cada divisor ai de n existe um 1 j r tal que n . logo ai = a j 1 1 1 + + ... + a1 a2 ar = = 1 1 1 1 (n + n + . . . + n ) n a1 a2 ar 1 1 (a1 + a2 + . . . + ar ) = .2n = 2 n n

Nota c ao: a b, a congruente a b m odulo k, isto e, a e b deixam mesmo resto na divis ao por k ou, equivalentemente, a b e divis vel por k. Arma c ao 10. Se N > 6 e um n umero perfeito par, ent ao N 1 e divis vel por 3. Demonstra ca o. Como N > 6 e N = 2p1 (2p 1), temos que, 2p 2,
3 3 3 3

donde conclu mos que, 2p 1 1 e 2p1 1, logo 2p1 (2p 1) 1. Portanto, 3 2p1 (2p 1) 1 ou seja N 1 e divis vel por 3.

Arma c ao 11. Todo n umero perfeito par termina em 6 ou 8. Demonstra ca o. Seja N = 2p1 (2p 1) = 2p1 2p 2p o n umero perfeito par em quest ao, se p = 2 temos N = 6. Vamos agora provar para p > 2, observe que 4 2, se n 1 4 4, se n 2 n 10 2 4 8, se n 3 4 6, se n 0. Como p um primo maior que 2 temos que p 1 e par, assim teremos, 2p1 4 4, se p 1 2 10 4 6, se p 1 0 4 8 se p 1 2 p 10 e 2 4 2, se p 1 0

Olimp ada de Matem atica do Estado de Goi as

107

donde conclu mos que 2 logo


p1 p 10

4 2, se p 1 2 2 4 2, se p 1 0

2p1 2p 2p1

Portanto, qualquer que seja o resto deixado por p 1 na divis ao por 4, teremos que o resto deixado por um n umero perfeito par na divis ao por 10 ser a 6 ou 8. Arma c ao 12. A soma interativa dos algarismos de um n umero perfeito par que n ao o 6 e 1, isto e, dado um n umero perfeito par se somarmos os seus algarismos em seguida somarmos os algarismos do resultado obtido e assim sucessivamente, teremos ao nal 1. Demonstra ca o. Primeiro vamos observar que a soma interativa de um n umero e simplesmente a sua classe de congru encia m odulo 9, pois se N = ar a1 a0 sendo ai com i {0, 1, 2, . . . , r } os d gitos do n umero N , ent ao
r

4 2 4 10 8, se p 1 2 10 4 2 6 10 6, se p 1 0.

N=
i=0

ai 10

i 9

ai (1) =
i=0 i=0

ai .
9

Basta agora mostrar que se N e perfeito par ent ao N 1. 2n1 (2n


3 3

Seja

como 26 1 segue que N 1(2 1) = 1. Agora no caso em que n 2, isto e, n = 3k2 + 2 para algum k2 natural, como n n ao e par, temos que k2 e mpar, portanto podemos escrever n = 6k + 5 onde k2 = 2k + 1.

N n ao e perfeito. Se n 1, isto e n = 3k1 + 1 para algum k1 > 0, como n e primo e N = 6, segue que n e mpar, conseq uentemente podemos es6 k 6 k +1 crever n = 3k1 +1 = 6k +1, assim N = 2 (2 1) = (26 )k [2(26 )k 1].
9 9 3

N= 1) um n umero perfeito par. Se n 0 ou seja 3 divide n teremos que n n ao e primo e portanto 2n 1 tamb em n ao e primo, assim

Olimp ada de Matem atica do Estado de Goi as

108

Logo N = 26k+4 (26k+5 1) 16(1)[32(1) 1] (2)[(4) 1] 10 1.


9 9 9 9

= 16(26 )k [32(26 )k 1]

Vejamos um exemplo. Exemplo 5. Tomemos o n umero perfeito 496. A soma de seus algarismos e 19, que por sua vez possui soma 10, cuja soma e 1. Arma c ao 13. Um n umero perfeito par N = 2n1 (2n 1) escrito na base 2 t em os n primeiros d gitos iguais a 1 seguidos de n1 zeros, isto e, N = (11 1 00 0)2 .
n n 1

Demonstra ca o. Sabemos que a soma 1 + 2 + 22 + + 2n1 = 2n 1, portanto N = 2n1 (2n 1) = 2n1 (1+2+22 + +2n1 ) = 2n1 +2n + +22n2 .

Veja a representa c ao de alguns n umeros perfeitos na base 2. 6 = (110)2 , 28 = (11100)2 , 496 = (111110000)2 . Com essa propriedade encerramos nossa colet anea e deixamos aqui, atrav es das refer encias indicadas abaixo, o convite ao leitor interessado neste assunto, para ler e se deliciar de outras belas propriedades que cercam os n umeros perfeitos e primos de Mersenne.

Bibliograa
[1] Souza, J. C. M. Matem atica Divertida e Curiosa, RECORD, RJ, 2002.

Olimp ada de Matem atica do Estado de Goi as

109

[2] Boyer, C. B. Hist oria da Matem atica, Ed. Edgard Blucher, 2a. edi c ao, trad. Elza E. Gomides, SP. 1999. [3] Davis, S. it A proof of the odd Perfect Number conjecture, Number Theory,(preprint) 2004. [arxiv.org/abs/hep-th/0401052; acesso em 25/09/2007] [4] Ribenboim, P. N umeros Primos: mist erios e recordes, Cole c ao Matem atica Universit aria,IMPA RJ. 1999. [5] Guy, R.K. Unsolved Problems in Number Theory, SpringerVerlar, New York, 1982. [6] Ore, O. Nunber Theory and this History,Dover Plublications, New York, 1988. [7] Domingues, H.H., Fundamentos de Aritm etica, Atual Editora, SP, 1988. [8] Alencar Filho,E., Fun co es Aritim eticas N umeros Not aveis,Atual Nobel, SP, 1988. [9] The Largest Known Primes [primes.utm.edu; 25/09/2007]. acesso em

[10] The digital sum of a perfect number of Euclids type is 1 [www-maths.swan.ac.uk/pgrads/bb/project/node7.html; acesso em 22/11/2007]. Autor: Endere co: Alacyr Jos e Gomes Universidade de Federal de Goi as Instituto de Matem atica e Estat stica Caixa Postal 131 74.001-970 Goi ania, GO, Brasil alacyr@mat.ufg.br Eudes Antonio da Costa Universidade de Federal do Tocantins Campus Universit ario de Arraias

Autor: Endere co:

Olimp ada de Matem atica do Estado de Goi as

110

Curso de Matem atica Av. Universit aria, s/n, Centro 77.330-000 Arraias, TO, Brasil eudes@uft.edu.br Autor: Endere co: Ronaldo Antonio dos Santos Universidade de Federal de Goi as Instituto de Matem atica e Estat stica Caixa Postal 131 74.001-970 Goi ania, GO, Brasil rasantos@mat.ufg.br

Revista da Olimp ada - IME - UFG, no - 7, 2006

Objetivo e Pol tica Editorial A Revista da Olimp ada tem como objetivo ser um ve culo de difus ao, principalmente, das Olimp adas de Matem atica do Estado de Goi as, promovidas pelo IME/UFG. A Revista tamb em est a aberta a contribui c oes de pequenas mat erias, subordinados ` a boa qualidade. O material submetido para a publica c ao dever a ser de interesse do Ensino Fundamental e M edio, estar bem redigido, em estilo claro, sem aridez, de forma que desperte o interesse do leitor. Submiss ao e Aceite Toda mat eria submetida para publica c ao deve ser enviada ao Comit e A Editorial. Mat erias redigidas em TEX ou L TEX podem ser submetidas por e-mail: omeg@mat.ufg.br. Se existirem ilustra c oes no trabalho submetido, estas devem ser encaminhadas, juntamente com o trabalho, e precisam estar em condi c oes de serem reproduzidas, sem retoques. Al em disso, c opias dos desenhos e ilustra c oes devem ser axadas em espa cos apropriados do texto, exibindo, dessa maneira, como dever a car a apresenta c ao nal do trabalho. As refer encias bibliogr acas devem ser colocadas no nal do texto, em ordem alfab etica, segundo as normas da ABNT. As mat erias submetidas para publica c ao ser ao analisadas pelos editores que poder ao solicitar pareceres ad hoc e o autor receber a a resposta sobre sua mat eria num prazo m aximo de 120 dias. Os autores que tiverem os trabalhos aceitos dever ao transferir seus direitos autorais para o Instituto de Matem atica e Estat stica da UFG.

S-ar putea să vă placă și